Вы находитесь на странице: 1из 209

SOLUCIONARIO

PROBLEMAS CURSO
PRODUCCION
Ingº Luis Del Castillo Rodríguez
2004

1
Cálculo Permeabilidades Relativas
Problema 1):
Cálculo de las permeabilidades de datos de
pruebas de Laboratorio de Núcleos.
Un núcleo ha sido tomado de una arenisca
reservorio, pruebas de permeabilidad han
sido realizadas en el laboratorio y los
siguientes datos han sido obtenidos:
Longitud del núcleo = 5 cm.
Diámetro del núcleo = 2.5 cm.
Presión de entrada cara del núcleo = 3 atm.
Presión de salida cara del núcleo = 1 atm.
Viscosidad del petróleo = 1.3 cp.
Viscosidad del agua = 1.0 cp.
2
Cálculo Permeabilidades Relativas
Se tienen los siguientes datos de rate y saturación:
Saturación Saturación Rate Flujo Rate Flujo
Petróleo Agua Petróleo Agua
% % cc/seg cc/seg
0 100 0.00 0.40
10 90 0.00 Crítica 0.31
30 70 0.01 0.16
50 50 0.15 0.05
60 40 0.25 0.01
70 30 0.30 0Crit.

3
Cálculo Permeabilidades Relativas
1.- Cálculo de la permeabilidad absoluta
Ecuación de Darcy :
q K DP
v= =-
A m DL
KA DP
q=
m DL
mw qw DL 1.0*0.4 5
k= = - * = 0.2037 Darcys
A DP p*2.5²/4 (1-3)
qw = 0.4 cc/seg a Sw = 100%
k = 203.7 md.
2.-Cálculo de las permeabilidades efectivas y
relativas
mw qw DL moqo DL
kw = ; Ko =
A DP A DP
4
Cálculo Permeabilidades Relativas
Donde todos los términos son constantes, excepto
los respectivos rates para cada saturación.
Kw = 0.50916 qw darcys
Ko = 0.6619 qo darcys

Sw % qw cc/seg Kw (darcys) Kr w = Kw / 0.2037

100 0.40 0.2037 1.0000


90 0.31 0.1578 0.7777
70 0.16 0.0815 0.4000
50 0.05 0.0255 0.1252
40 0.01 0.0051 0.0250
30 0.00 0.0000 0.0000

5
Cálculo Permeabilidades Relativas
So % qo cc/seg Ko (darcys) Kro = Ko / 0.2037

0 0.00 0.0000 0.0000


10 0.00 0.0000 0.0000
30 0.01 0.0066 0.0324
50 0.15 0.0993 0.4875
60 0.25 0.1655 0.8125
70 0.30 0.1986 0.9748
3.- Ploteo de la curva permeabilidad relativa vs.
Saturación de agua.

6
Cálculo Permeabilidades Relativas

7
Permeabilidades pseudorelaivas
Problema 2): Preparar la curva de permeabilidad
pseudorelativa para un reservorio de 5 capas,
asumiendo avance de agua de fondo. Cada capa
tiene 10' de espesor y el contacto agua-petróleo
está inicialmente en la base de la capa inferior. Del
fondo al tope las permeabilidades de la arena son
50,500,1500,2000 y 500 md. Se asume que en cada
capa las siguientes propiedades se aplican:
Permeabilidad relativa al petróleo a la saturación de
agua inicial de 15% es 0.9.
Permeabilidad relativa al agua a la saturación de
petróleo residual de 30% es 0.5.
Para cada capa: Swi = 0.15; So r= 0.3; k'ro = 0.9; k'rw =
0.5.
8
Permeabilidades pseudorelaivas
5
Solución: Si hacemos S kj = R = 4550
1

n N n N _ _ _
n kj Skj Skj (Skj)/R (Skj)/R krwn kron Swn
1 n+1 1 n+1

-------------------------------------------------------------------------
0 0 0 4550 0 1.00 0 .9 .15
1 50 50 4500 .0110 .989 .0055 .8901 .26
2 500 550 4000 .1209 .879 .0605 .7911 .37
3 1500 2050 2500 .4505 .5494 .2253 .494 .48
4 2000 4050 500 .8901 .1099 .4451 .0989 .59
5 500 4550 0 1.000 0 .5 0 .7

9
Permeabilidades pseudorelaivas
Fórmulas:
n 5
0.5Skj 0.9 S kj
1 n+1

krwn= ; kron=
4550 4550
Swn = (0.7 n + 0.15(5-n))/5
Las permeabilidades pseudorelativas resultantes
son ploteadas como Swn vs Krw y kro.

10
Permeabilidades pseudorelaivas

11
Presión Capilar
PROBLEMA 3): Un experimento ha sido realizado
para determinar la presión capilar de un núcleo
mediante la inyección de mercurio.
Tensión superficial del mercurio = 480 dinas/cm.
Angulo de contacto del mercurio = 140
Tensión interfacial agua-petróleo, reservorio = 28
dinas/cm. Angulo de contacto de reservorio = 0
En la tabla las columnas 1 y 2 son datos y la 3 es la
presión capilar a condiciones del reservorio.
(1) (2) (3) (4)
Presión inyec- Saturación Presión capilar Altura desde
ción mercurio de agua reservorio el contacto
agua-petróleo
psia % psia pies
11.8 100.00 0.90 14.25
13.0 85.57 0.99 15.68
13.8 75.58 1.05 16.63
15.1 55.60 1.15 18.21

12
Presión Capilar
(1) (2) (3) (4)
Presión inyec- Saturación Presión capilar Altura desde
ción mercurio de agua reservorio el contacto
agua-petróleo
psia % psia pies
17.0 47.83 1.30 20.59
20.3 43.39 1.55 24.55
21.0 42.28 1.60 25.34
25.0 40.01 1.90 30.09
27.0 37.74 2.06 32.62
31.8 35.62 2.42 38.32
32.8 34.51 2.50 39.59
38.0 31.21 2.90 45.93
42.0 30.02 3.20 50.68
46.0 28.96 3.50 55.43
55.0 26.74 4.19 66.35
63.0 24.72 4.80 76.01
80.0 22.01 6.09 99.44
112.0 21.10 8.53 135.08
13
Presión Capilar
La columna (3) se obtuvo de:
Pc Res = ((Cosq)Res /(Cosq)Lab)* Pc Lab
Pc Res = (28x1/480*0.766) * columna1
= 0.076 * Columna(1)
La columna (3) presión capilar vs. columna (2)
saturación de agua, ha sido ploteada en la Figura .
Deduciéndose que la saturación de agua irreductible
es 21%.
Utilizando la definición inicial de presión capilar:
Pc = 0.433* D gravedades especificas (agua-fluido)* h;
gradiente del agua = w = 0.433 psi/ft
h = altura en el capilar del fluido que moja
Los datos de la columna (3) de presión capilar en el
reservorio pueden ser dados como alturas, a partir de
la intersección de la gradientes de los fluidos
(contacto agua-petróleo por gravedad). 14
Presión Capilar
Si rw= 65.3 y ro= 56.2 lb/ft3 a cond. res.
Pc Res = 0.433[(65.3-56.2)//62.4]*h;
h = Columna (3)x15.836
Así hallamos la columna (4), y al plotear las columnas
(4) vs la (2), nos indica una zona de transición de 130'
Hasta aquí se ha considerado un ángulo de contacto
en el reservorio de 0, muy difícil encontrar una roca
totalmente mojada en agua, de manera que las
alturas dadas en la columna (4) son ideales.
La relación de las alturas reales de registros eléc-
tricos, para identificar saturaciones con las alturas
ideales ya calculadas, permite hallar qreal
Pc real (Cosq)real 0.433 hreal hreal
= = =
Pc ideal (Cos0)ideal 0.433 hideal hideal
Si en el ejemplo ese cociente es un promedio de 0.85,
el ángulo de contacto y la mojabilidad es de 32. 15
Presión Capilar

16
Presión Capilar
PROBLEMA 4): Dados los datos de curvas de
presión capilar de la Figura, correspondientes a una
formación. Realizar el ploteo de saturación de agua
vs. profundidad para este reservorio. El 100% de
saturación de agua, fue determinado del análisis de
núcleo a 4053'. La densidad del agua y del petróleo
a condiciones del reservorio, son 65.3 y 56.2 lb/ft3,
respectivamente.
Zona Profundidad Permeabilidad Porosidad
Pies promedio md promedio
I 3998-4007' 564 0.272
II 4007-4019' 166 0.208
III 4019-4031' 591 0.273
4031-4037' Shale
IV 4037-4046' 10.2 0.142
V 4046-4055' 72 0.191

17
Presión Capilar
Solución:Como el nivel del contacto fue encontrado
a 4053', éste se encuentra en la zona V y para esta
zona en la Figura, la presión de entrada es 0.45 psi.,
luego el corte de las gradientes estará a:
0.45 = 0.433[(65.3-56.2)/62.4]h ; h = 7.13' = 7'
Nivel agua libre = 4053' + 7' = 4060'
Con la fórmula Pc = 0.063h, donde h son alturas por
encima de 4060‘, obtenemos Pc y las saturaciones
de las respectivas curvas de Pc de la Figura.
Profundidad Altura sobre Presión Zona Sw
Pies 4060’ nivel capilar
Agua libre psi %__
4053 7 0.45 V 100
4050 10 0.63 V 60
4046 14 0.88 V 45
18
Presión Capilar

19
Presión Capilar
Profundidad Altura sobre Presión Zona Sw
Pies 4060’ nivel capilar
Agua libre psi %__
4045 15 0.94 IV 80
4042 18 1.07 IV 67
4039 21 1.33 IV 53
4037 23 1.45 IV 49
4036 Shale 100
4033 Shale 100
4031 29 1.83 III 19
4025 35 2.21 III 17
4021 39 2.46 III 16
4019 41 2.58 III 15
4018 42 2.65 II 23
4009 51 3.21 II 22
4006 54 3.40 I 16
4000 60 3.78 I 16
La distribución de saturaciones del reservorio encontrada la
tenemos ploteada en la Figura.

20
Perfil Saturación de Agua

21
Presión Capilar
Problema 5): Curvas de presión capilar, fueron dadas
sobre 75 muestras tomadas al azar de diversos
intervalos de una arena-reservorio coreada. Cinco de
esas curvas son dadas en la Figura:
Otros datos obtenidos, son los que siguen:
ow= tensión interfacial, petróleo-agua a condi-
ciones del reservorio = 33 dinas/cm.
w = tensión superficial, agua-aire, a condi-
ciones de la prueba = 71 dinas/cm.
w = gradiente agua a condiciones reservorio =
0.45 psi/pie.
o = gradiente petróleo condiciones reservorio =
0.31 psi/pie.
kav = permeabilidad promedio absoluta para el
reservorio obtenida de 1,000 muestras
analizadas = 155 md.
22
Presión Capilar

23
Presión Capilar
Encontrar:
1) Correlaciones entre k y saturación de agua.
2) Curva de Pc promedio para el reservorio.
3) Factor de conversión para relacionar presión
capilar con elevación desde el subsuelo sobre el
nivel de contacto por gravedad.
4) Saturación de agua irreductible.
Solución:
1) La Figura, muestra 5 curvas típicas de presión
capilar de las 75 involucradas en este problema.
Cada curva, fue obtenida de una muestra
individual, la permeabilidad está anotada sobre
cada curva. Usando presiones capilares de
75,50,25,10 y 5 psi, la saturación de agua fue leída
para cada curva. La tabla 1, muestra los valores
leídos para las presiones capilares señaladas.
24
Gráfico semilog k vs Sw

25
Presión Capilar
Tabla 1.-Permeabilidad vs Saturación de Agua
a Presión Capilar Constante
Permea- Saturación de Agua a Presión Capilar Constante
bilidad 75psi 50psi 25psi 10psi 5psi
950.00 14.00 16.50 22.00 30.00 39.00
300.00 22.50 25.50 34.00 45.50 56.00
50.00 30.00 36.00 47.00 61.50 78.00
25.00 44.50 47.50 59.00 71.50 83.00
10.00 50.50 53.00 63.50 81.00 92.00
Una recta es trazada a través de un gráfico semilog, per-
meabilidad vs. saturación de agua, eje log (k).
2)De datos de análisis de cores, se halla un k promedio
del reservorio, usado para obtener valores de la Pc del
reservorio, del gráfico anterior y ploteados en el
siguiente gráfico.
Tabla 2.-Presión Capilar Promedia del Reservorio kav = 155 md
Pc psi Sw % Pc psi Sw %
75 27 10 51
50 30 5 62
25 39
26
Presión Capilar

27
Presión Capilar
3)Correlaciones entre presión capilar y
distancia del nivel de contacto por
gravedad, está dado por la ecuación:
w 71
PcL = Dow h = (.45-.31)h
ow 33
h = 3.3 PcL
h es la correspondiente distancia vertical en
el subsuelo desde el nivel de agua libre(al
final de la zona de transición, ó desde el
nivel 100% de saturación de agua).
4)Del último gráfico se nota, que la
saturación de agua irreductible es 26.5%.
28
Presión Capilar
Problema 6): Para una formación productiva, el
ploteo de la función J de Leverett, es el siguiente:
Sw J Sw J
100 0.35 30 1.45
70 0.40 20 3.15
54 0.45 15 6.15
44 0.60
De análisis de núcleos, se tiene lo siguiente:
Zona Prof.-Pies Permeabilidad Porosidad
____ Promedio,md. Promedio md _______
I 5,995'-6,003' 15 0.142
II 6,003'-6,015' 8 0.107
6,015'-6,031'Lutitas
III 6,031'-6,049' 16 0.143
IV 6,049'-6,055' 5 0.101
V 6,055'-6,063' 10 0.123
29
Presión Capilar
a) Si la gradiente del petróleo es 0.35 psi/ft y del
agua 0.45 psi/ft y una prueba a 6010' en la zona
de petróleo dio una presión del reservorio de
2725 psia, encontrar el contacto agua-petróleo
por gravedad.
b) Si la tensión interfasial en el reservorio
agua-petróleo es 28 dinas/cm y el contacto agua
petróleo de los registros eléctricos se encuentra
a 6060.4', encontrar la mojabilidad.
c) Trazar las curvas de presión capilar de cada zona
y dibujar el perfil de saturaciones de agua.
d)Si la permeabilidad promedio de la formación es
12 md trazar la curva de presión capilar
promedia.
Solución:
a) 6,010*0.35 + C1 = 2,725
C1 = 2,725 - 6,010 * 0.35 = 621.50
30
Presión Capilar
D*0.45 + 14.7 = D*0.35 + 621.5
D = (621.5-14.7)/0.10 = 6068‘
b) El ploteo de la función J de Leverett, es el
siguiente:
h = 6,068-6,060.4 = 7.6'
Pc = 0.1*7.6 = 0.76
Cosq = (10/0.123)0.5*0.76 / (0.35*28) = 0.7
q = 45.63
c) Zona I y III, Pc = J*28*0.7 / 10.4 = 1.9J
Zona II, Pc = J*28*0.7 / 8.65 = 2.27J
Zona IV, Pc = J*28*0.7 / 7.04 = 2.78J
Zona V, Pc = J*28*0.7 / 9.02 = 2.17J

31
Presión Capilar
Presión Capilar
Sw J I y III Zona II Zona IV Zona V
----- ------ --------- -------- -------- ---------
100 0.35 0.67 0.79 0.97 0.76
70 0.40 0.76 0.91 1.11 0.87
54 0.45 0.86 1.02 1.25 0.98
44 0.60 1.14 1.36 1.67 1.30
30 1.45 2.76 3.29 4.03 3.15
20 3.15 5.98 7.15 8.76 6.84
15 6.15 11.68 13.96 17.10 13.35

32
Curvas de Presión Capilar

33
Presión Capilar
Perfil de Saturación de Agua
Profundidad Saturación Profundidad Saturación
Pies Agua Pies Agua
--------------------------------------------------------------------------
6060 100 6031 27
6058 55 6031 100
6056 47 6015 100
6055 44 6015 24
6055 54 6010 23
6053 48 6007 22
6051 43 6003 22
6049 42 6003 20
6049 37 6000 19
6047 36 5998 19
6040 29 5995 19
6035 28

34
Perfil Sw

35
Perfil de gradientes
Problema 7): Un pozo perforó la formación produc-
tiva encontrando gas hasta 5502'. La presión en el
reservorio a 5467' fue de 3098 psia. Si el agua esta
sobrepresurizada en 410 psia sobre la presión hi-
drostática normal. Donde debe estar el contacto de
agua para que no halla anillo de petróleo. Cual es el
tamaño del anillo si el nivel de agua está a 7100'.
g = 0.089 psi/ft ; w = 0.45 psi/ft ;o = 0.35 psi/ft
Pg = 0.089x5467 + C2 = 3098 psia; C2 = 2611.437
Pg = 0.089xD + 2611.437 = 0.45xD + 424.7
D = 6057.47' (sin anillo de petróleo).
Po = 0.35x7100 + C1 = 0.45x7100 + 424.7;C1 = 1134.7
Pg = 0.089xD + 2611.437 = 0.35xD + 1134.7
D = 5658'
anillo de petróleo = (7100-5658) = 1442'
36
Perfil de gradientes

37
Perfil de gradientes
Problema 8): Un pozo perforó la formación pro-
ductiva encontrando gas hasta 8200'. La pre-
sión a 8000' fue de 3800 psia. Buzamiento 20.
1.-¿Cuáles serán los niveles de agua para tener
la mínima y máxima columna de petróleo?
2.-Si el posible nivel de agua está a 8800 pies, a
que distancia mínima se perforará un según-
do pozo para probar si existe petróleo.
g=0.08psi/pie;o=0.35 psi/pie;w= 0.45 psi/pie
Solución:
1) Niveles máxima y mínima columna de aceite
Pg = 0.08*8000 + C2 = 3800 psia; C2 = 3160
Si hubiera petróleo desde 8200 pies
Pg = 0.08*8200 + 3160 = 0.35*8200 + C1 = Po
C1 = 946
38
Perfil de gradientes
Po = 0.35*D+ 946 = 0.45*D + 14.7 = Pw; D = 9313'
Si no hubiera petróleo el contacto gas/agua:
0.08*D+ 3160= 0.45*D + 14.7
D = 8500'
Para máxima columna de petróleo nivel de agua
9313', mínima columna nivel agua 8500'
2) Posible nivel de agua = 8800'
Ecuaciones del gas y aceite iguales en el contacto:
0.45 * 8800 + 14.7 = 0.35 * 8800 + C1
C1 = 894.7
El contacto gas/petróleo estaría :
0.35 * D + 894.7 = 0.08 * D + 3160
D = 8390'
Distancia mínima = (8390 - 8000 ) *Ctg 20 = 1071.5'

39
Perfil de gradientes

40
Factor de Volumen y Rsi
Problema 9): Experimentos han sido realizados con
una muestra de petróleo del fondo de un pozo, para
determinar: (a) El gas en solución y (b) El factor de
volumen de formación, como función de la presión.
La presión inicial del reservorio fue de 3600 psia. y
la temperatura de fondo 160F; todas las medidas
en el laboratorio fueron también hechas a 160F.
Los siguientes datos fueron obtenidos.
Presión Gas Solución Factor Volumen
psia SCF/STB Formación bo
Bbl/STB
3600 567 1.31
3200 567 1.317
2800 567 1.325
2500 567 1.333
41
Factor de Volumen y Rsi
Presión Gas Solución Factor Volumen
psia SCF/STB Formación bo
Bbl/STB
2400 554 1.31
1800 436 1.263
1200 337 1.21
600 223 1.41
200 143 1.07
La presión de burbuja puede ser encontrada por el
cambio de pendiente del factor de volumen del pe-
tróleo que ocurre a 2500 psia. En la condición inicial
3600 psia el petróleo está bajo saturado, que se
constata por haber alcanzado un valor constante de
gas en solución.
42
Factor de Volumen y Rsi

43
Factor de Volumen Compuesto
Problema 10): Calcular el factor de volumen de
formación total para un sistema de gas y
condensado, a condiciones del reservorio, dada la
siguiente información:
Presión del reservorio = 3000 psia.
Temperatura del reservorio = 250ºF
Producción de condensado = 325 STB
ºAPI condensado a Cond. Stand.= 45º
Rate de vapor en el tanque = 170 MSCF
Gravedad del vapor = 1.2
Rate de gas en el separador = 3700 MSCF
Grav. del gas en el separador = 0.65
Solución: Base = 1 Bbl condensado a Stock-Tank.

44
Factor de Volumen Compuesto
Gravedad promedia del gas en superficie:
3.7*0.65 + 0.17*1.2
gg = = 0.674
3.7 + 0.17
325
STB condensado / MMSCF = = 84
3.7 + 0.17
Del gráfico, entrando con 84 y cortando la gg y API
del crudo: g del fluido del pozo / gg = 1.367
g del fluido del pozo = 1.367*0.674 = 0.921
Del gráfico, con la g del fluido, cortamos las curvas
para condensados.
Tpc = 432 R Ppc = 647 psia.
Del gráfico de compresibilidades: Z = 0.845

45
Factor de Volumen Compuesto
Ahora calculamos el número de moles de
condensado y gas por barril de condensado.
350 * gcond.
Nº de moles de condensado =
PM cond
gcond = 0.8
Peso molecular del condensado, del gráfico
entramos con el API del crudo.
PMcond = 140 lb/mol
350 * 0.8
#moles condensado = = 2 moles/stbcond
140
3.87x10 scf moles
#moles gas = = 31.4
379 scf/mol*325 stbcon stbcon
n = 31.4 + 2 = 33.4
46
Factor de Volumen Compuesto
De la ley de los gases:
nZRT 33.4*0.845*10.73*710 SCF
V= = = 71.7
P 3000 STBcond
71.7 Bbl
Bt = = 12.8
5.615 STBcond

47
IPR Pozo Reservorio bajosaturado
Problema 11): Usando la Ley de Darcy para
flujo seudoestable (flujo líquido de una sola
fase).
Datos: ko= 10 md; h = 30' ; API = 36; Tavg =
180F; espaciamiento = 80 acres romboidal;
gg = 0.7 ; PR = 2500 psia; hp = 20'; GOR= 400
scf/stb ; s= 4; diámetro del hueco perforado
= 10 3/4"
Calcular:
(I) Rate del pozo a la presión de burbuja.
(II) Indice de productividad del pozo.

48
IPR Pozo Reservorio bajosaturado
Solución: (I) Rate del pozo a la presión de
2000 psia.
(1) Determinar la presión de burbuja,
mediante la ecuación:
100.00091T
Pb = [GOR/gg]0.83 { 18x } = 1808 psia
100.0125API
T= F
(2) Determinar bo a la presión de burbuja.
bo = 0.972 + 0.000147 F1.175
F = GOR(gg/go)0.5 + 1.25T ; F = 589.11
bo = 1.2364
49
IPR Pozo Reservorio bajosaturado
(3) Determinar mo a la presión de burbuja.
(a) De correlación, m od = 1.5 cp.
(b) Corregido por el GOR, mo = 0.735 cp.
(4) Determinar x para un pozo en el centro
de un rombo de 80 acres. De la Fig.2.9.
0.565 A½
x = ---------- = 2344 ; ln 0.472x = 7.02
rw
(5) Determinar a:
141.2 mo bo [ln 0.472x+s]
a = ---------------------------------- = 4.717
kh

50
IPR Pozo Reservorio bajosaturado
(6) Determinar b:
9.08x10-13 b bo²r 2.33x1010
b = ----------------------- ; b= ------------ = 1.47x109
4p² hp² rw k1.201
b = 1.52x10-5
(7) Sin turbulencia
(Pr-Pwf) = 691.365 = 4.717 q
q = 146.68 b/d
(7) Escribir la ecuación:
(Pr-Pwf) = 691.365 = 4.717 q + 1.52x10-5 q2
q = 146.49 b/d
II) índice de productividad
J = 146.49/691.365 = 0.212 b/d/psi.

51
IPR Pozo Reservorio bajosaturado

52
Pozo en Infinite Acting
Problema 11a): Flujo de una sola fase. Pi =3250 psi
f = 0.14 gg = 0.65 mo = 0.8807 cp re = 1500’ h= 70’
ct = 2E-6 psi-1 T = 175ºF ko = 1 md API 30º
bo =1.2586 rb/stb rw = 0.33’ s = 0. Calcular:
1)El comienzo del período pseudoestable.
2)Plotear el rate de flujo vs tiempo, para presión
constante de fondo fluyente de 2000 psi.
3)Construir la curva IPR para Infinite Acting.
Solución: re2 948fmct re2
1) t= = = 526 horas
4 k
2) Ecuación Flujo (Estado Transitorio):
0.388Dp 485.48
q= =
log t  4.34 log t  4.34
53
Pozo en Infinite Acting
t hr Rate bl/d Tiempo hr Rate bl/d
0 0 250 72.05
1 111.86 300 71.21
10 90.91 350 70.52
50 80.39 400 69.93
100 76.57 450 69.42
150 74.50 500 68.97
200 73.10 526 68.75
Ecuación Flujo (Estado Pseudoestable), cuando
Dp = pi – pwf ;
0.000527 kt re
pD =  ln  3/ 4
fm c t re2 rw

q = 0.0547 Dp (recta en cartesiano)


El rate al pseudoestable es 68.4 bl/d.

54
Pozo en Infinite Acting

55
Pozo en Infinite Acting
3) IPR Horas
Pwf t=10 t=50 t=100 t=200 t=350 t=526
3000 18 16 15 15 14 14
2800 33 29 28 26 25 25
2600 47 42 40 38 37 36
2400 62 55 52 50 48 47
2200 76 67 64 61 59 58
2000 91 80 76 73 70 69
1800 105 93 89 85 82 80
1600 120 106 101 96 93 91
1400 134 119 113 108 104 102
1200 149 132 125 120 116 113
1000 163 145 138 131 127 124
800 178 157 150 143 138 135
600 193 170 162 155 149 146
400 207 183 174 167 161 157
200 222 196 187 178 172 168
0 236 209 199 190 183 179
56
Pozo en Infinite Acting

57
Pozo en Infinite Acting
Problema 11b): Flujo de una sola fase. Pi = 2000 psi;
f = 0.15; gg = 0.65; mo = 0.4 cp; re = 1300’; h = 50’
ct = 2E-5 psi-1 ;T = 175ºF ; ko = 5 md ; API 30º
bo =1.2 rb/stb; rw = 0.25; s = 2. Calcular:
1) El comienzo del período pseudoestable.
2) Plotear el rate de flujo vs tiempo, para presión
constante de fondo fluyente de 1200 psi.
3) Construir la curva IPR para Infinite Acting.
Solución:
1) re2 948 fm c t re2
t= = = 384 .5horas
4 k
t hrs PD Rate bl/d t hrs PD Rate bl/d
1 5.29 404.56 200 7.94 296.84
10 6.4 349.45 250 8.05 293.55
50 7.25 319.07 300 8.14 290.91
100 7.59 307.55 384.5 287.4

58
Pozo en Infinite Acting

59
Pozo en Infinite Acting
2) Ecuación Flujo (Estado Transitorio):
3.2Dp 2560.86
q= =
log t  6.33 log t  6.33
Flujo pseudoestable:q= 0.3578 Dp (recta cartesiano)
El rate al pseudoestable 286.25 bl/d
3) IPR al Infinite Acting
Pwf t = 10 t = 50 t = 100 t = 200 t =300 t= 384.5
1800 87 80 77 74 73 72
1600 175 159 154 148 145 144
1400 262 239 230 222 218 215
1200 349 319 307 297 291 287
1000 437 399 384 371 363 359
800 524 478 461 445 436 431
600 611 558 538 519 509 503
400 698 638 615 593 581 574
200 786 717 691 667 654 646
100 873 797 768 742 727 718
60
Pozo en Infinite Acting

61
Reservorio Saturado
Problema 12): Flujo de dos fases. Datos:Pr = 1600
psia; Pb = 1600 psia; qo = 150 b/d a un Pwf = 1000
psi. Determinar qomax y la curva IPR.
Solución: (1) De la ecuación de Vogel
qo/qmax = 1 - 0.2(Pwf/Pr) - 0.8(Pwf/Pr)²
qmax = 267 b/d.
(2)Determinación del IPR. Despejando de la
ecuación de Vogel _____________
Pwf = 0.125 Pr [-1+  81-80(qo/qomax) ]
con lo cual se tiene la siguiente Tabla:
qo Pwf qo Pwf qo Pwf
0 1600.00 100 1228.81 200 718.15
20 1532.14 120 1142.31 220 576.72
40 1461.51 140 1049.84 240 402.99
60 1387.73 160 949.96 267 0.00
80 1310.36 180 840.53
62
Reservorio Saturado

63
Reservorio bajosaturado
Problema 13): Datos: ko= 30 md ; h = 60' ; re = 2000' ;
PR = 3000 psia; s = 0 ; rw = 0.4' ; Pb = 2000 psia ; bo =
1.2; mo = 0.68 cp
Calcular: (I) rate del pozo a la presión de burbuja.
(II) rate máximo de producción.
(III) qo para pwf de 2500 y 1000 psia.
Solución: (I) Empleando la Ley de Darcy
7.08x10-3 koh (PR-Pb)
q = ---------------------------------- = 2010.72 b/d
mo bo (lnx - 3/4+ s+ Dq)
(II) qb = J(Pr-Pb); J = 2.011 b/d/psi
qmax = qb + (J Pb/1.8) = 4245.32 b/d.
(III) qo para pwf = 2500 psi
qo = J(Pr-Pwf) = 2.011(3000-2500)= 1005.5 b/d
qo para pwf = 1000 psi
qo = qb + (qmax-qb) [1-0.2(Pwf/Pb) - 0.8(Pwf/Pb)²]
qo = 3574.94 b/d 64
Reservorio bajosaturado
Problema 14): Datos: Pr = 4000 psia
Pb = 3000 psia qo = 600 para Pwf = 2000 psia.
Calcular: (1) J ; (2) qb ; (3) qmax ; (4) q para pw f= 3500
y 1000 psia.
(1) Determinación de J
q
J = ----------------------------------------------------------------
{Pr - Pb + (Pb/1.8) [1-0.2(Pwf/Pb)-0.8(Pwf/Pb)²]}
J = 0.324 b/d/psi
(2) qb = J(Pr-Pb) = 324 b/d
(3) qmax = qb +(J Pb/1.8) = 864 b/d
(4) q para pwf 3500 psia
qo = J(Pr-Pwf) = 162 b/d
q para pwf 1000 psia
q = qb+ [qmax-qb][1-0.2(Pwf/Pb)-0.8(Pwf/Pb)²]
q = 324 + 540 (0.844) = 780 b/d.
65
Reservorio bajosaturado
Problema 15): Haciendo uso de la Fig.2.25 prepa-
rada de una ecuación de la forma: q = J'o(Pr²-Pwf²)
Datos: Pr = 2000 psi EF = 2
qo = 1200 para para Pwf = 500 psia.
Calcular: (1) qmax para EF=1 ;(2) qmax para EF=2;(3) q
para pwf 1000 y EF de 2 y 2.5.
Solución:
(1) Pwf/Pr = 500/2500 = 0.25
De la curva EF=2, Fig.2.25
qo/qmax= 1.435
1200/qmax = 1.435
qmaxEF=1 = 1200/1.435 = 836 b/d.
(2) De la curva EF=2, Fig.2.25 para Pwf/Pr =0
qo/qmax= 1.53
qomax para EF = 2 = 1.53x836 = 1280 b/d

66
Reservorio bajosaturado
(3) qo para pwf 1000 y EF = 2.0
1000/2000 = 0.5
De la curva EF=2, Fig.2.25 para Pwf/Pr =0.5
qo/qmax= 1.165
qo = 1.165x836 = 974 b/d.
qo para pwf 1000 y EF=2.5
1000/2000 = 0.5
De curva EF = 2.5, Fig.2.25 para Pwf/P r = 0.5
qo/qmax = 1.4
qo = 1.4x836 = 1170 b/d.
Problema 15a): Una prueba en un pozo de petróleo,
arrojó 150 b/d. Presión del reservorio = 2000 psia.
Presión de Fondo Fluyente = 1500 psia EF=2.
Reservorio saturado. Encontrar y graficar el IPR del
pozo.
Solución: Se prepara la Tabla:
67
Reservorio bajosaturado
Pwf Pwf’ qo 1 – (Pwf/Pr)2
2000 2000
1800 1600 70.3 0.19
1600 1200 126.9 0.36
1400 800 169.7 0.51
1200 400 198.9 0.64
Pwf’ = Pr –EF(Pr-Pwf)
De la prueba Pwf = 1500 psia Pwf’ = 1000 psia
Aplicando Vogel : qomax = 214.3 b/d.
La columna qo de la Tabla, se obtiene de la ec.:
qo = 214.3 [1 - 0.2(Pwf’/Pr) - 0.8(Pwf’/Pr)2]
Ploteamos en log-log las dos últimas columnas de
la Tabla, para obtener la Ecuación de Fetkovich
Del gráfico obtenemos qmax =302 b/d y n = 0.8725
La Ecuación es q = 302 (1 – (Pwf/Pr)2)0.8725
68
Reservorio bajosaturado

69
Reservorio bajosaturado
La Tabla del IPR es:
Pwf q b/d Pwf q b/d
2000 0 800 259.38
1800 70.91 600 278.14
1600 123.85 400 291.43
1400 167.83 200 299.36
1200 204.60 0 302.00

70
Reservorio bajosaturado

71
Reservorio saturado
Ejemplo 2-5 A - pag 29
pr = 2000 psig ; pb = 2080 psig; EF =0.6
qo = 80 STB/d
pwf = 1500 psig
EF = 0.6 EF = 1 EF = 1.2
1-pwf/pr =0.25
Presión J* = 0.17 J* = 0.29 J* = 0.34
2000 0 0 0
1800 33 54 65
1765 39 64 75
1600 65 104 121
1500 80 127 147
1300 108 169 191
1000 149 222 243
72
Reservorio saturado
EF = 0.6 EF = 1 EF = 1.2
1-pwf/pr =0.25
Presión J* = 0.17 J* = 0.29 J* = 0.34
700 184 264 278
300 225 302
0 251 317 341
Restricción = 333.33 psig

73
Reservorio saturado

74
Reservorio bajosaturado
Ejemplo 2-5 B - pag 29
pr = 4000 psig ; pb = 2000 psig; EF =0.7
qo = 378 STB/d
pwf = 1200 psig
EF = 0.7 EF = 1 EF = 1.4
1-pwf/pb =0.4
Presión J* = 0.14 J* = 0.2 J* = 0.28
3750 35 50 70
3500 70 100 140
3250 105 150 210
3000 140 200 280
2750 175 250 350
2500 210 300 420
2250 245 350 490
75
Reservorio bajosaturado
Presión 1-pwf/pb EF = 0.7 EF = 1 EF = 1.4
2000 0 280 400 560
1750 0.125 314 447 624
1500 0.25 344 489 678
1250 0.375 373 525 721
1200 0.4 378 531 728
1000 0.5 398 555 753
750 0.625 421 580 774
500 0.75 441 600
250 0.875 458 614
0 1 473 622 816
Restricción = 571 psig

76
Reservorio bajosaturado

77
Reservorio bajosaturado dañado
Pr= 2000 psi; EF= 0,8; qo= 180 b/d ;Pwf=1400 psi;Pb= 1800 psi
(1-Pwf/Pb) = 0,222; JEF=.8= 0,317; JEF=1.3= 0,515; qomaxEF=1Vogel=
395,851 b/d ;qomaxEF=1,3Vogel= 440,503 b/d; qomaxEF=1,3= 543,425
b/d; Restricción = 571,429 psi
qo para EF=
Presión (1-Pwf/Pb) 0,8 1 1,3
J*= 0,317 0,396 0,515
2000 0 0,000 0,000 0,000
1800 200 63,336 79,170 102,921
1600 0,111 124,170 154,431 199,235
1400 0,222 180,000 221,872 282,335
1200 0,333 230,825 281,494 352,220
1000 0,444 276,646 333,297 408,890
800 0,556 317,463 377,281 452,346
600 0,667 353,275 413,445 482,587
400 0,778 384,083 441,790
200 0,889 409,887 462,315
0 1 430,686 475,022 543,425
78
Ploteo de Fetkovich
Problema 16): Prueba de flujo después de flujo para
un pozo de petróleo: Pr = 3000 psia Pb= 3000 psia
Prueba qo b/d Pwf psia Prueba qo b/d Pwf psia
1 80 2987 3 400 2863
2 200 2949 4 1000 2449
Determinar la ecuación de flujo y el AOFP.
Solución: Se prepara la siguiente Tabla:
qo b/d (Pr²-Pwf²)/1000 qo b/d (Pr²-Pwf²)/1000
---------- ------------------- -------- -------------------
80 77.83 400 803.23
200 303.40 1000 3002.40
Luego se grafica en log-log:
(a) El exponente n = 1/pendiente
n = 0.6928
(b) Se determina C
C = 3.879
79
Ploteo de Fetkovich
(c) La ecuación será:
q= 3.879((Pr²-Pwf²)/1000)0.6928
(d) El AOFP será:
qmax = 2129.2 b/d.
La Ecuación de Fetkovich, resulta ser:
q= 2129.2(1-(Pr/Pwf)²)0.6928
qo b/d 1- (Pr/Pwf)²
---------- --------------
80 0.008648
200 0.033711
400 0.089248
1000 3

80
Ploteo de Fetkovich

81
Ploteo de Fetkovich

82
Ploteo de Fetkovich
Problema 16): Prueba de flujo después de flujo para
pozo de petróleo: Pr = 3600 psia. Datos de prueba:
qo STB/d Presión psia (pr2-pwf2)/106
263 3170 2.9111
383 2897 4.567391
497 2440 7.0064
640 2150 8.3375
n = 0.80109895
log C = 2.04757744
C = 111.577709
AOF = 868.728643 b/d

83
Ploteo de Fetkovich
Presión qo Presión qo
psia STB/d psi STB/d
3600 0 1500 746
3300 200 1200 791
3000 336 900 825
2700 448 600 849
2400 543 300 864
2100 623 0 869
1800 690

84
Ploteo de Fetkovich

85
Ploteo de Jones, Blount y Glaze
Problema 17): Hacer el ploteo de Jones, Blount y
Glaze de una prueba de cuatro puntos para un pozo
de petróleo: Pr = 1200 psia
Prueba N q Pwf
------------- ------- --------
1 70 1147
2 147 1023
3 209 856
4 292 530
Calcular:(1) a y b.
(2) qomax o AOFP.
(3) recomendaciones para mejorar la completación.
Solución:
(1) Preparar la tabla de q vs Dp / q
86
Ploteo de Jones, Blount y Glaze
q Pwf Dp / q q Pwf Dp / q
------- -------- ---------- ------ ------ ---------
70 1147 0.7571 209 856 1.6459
147 1023 1.2041 292 530 2.2945
(2)Determinar la pendiente y la intersección como
se anota en la Figura.
3)Se determina la pendiente y la intersección con la
ordenada. a= 0.2475 b= 0.00693
(4)Se encuentra qomax
1200 = 0.2475 q + 0.00693 q²; qomax = 398.65 b/d
(5)Se calcula a'
a' = 0.2475 + 0.00693x398.65 = 3.01
a'/a = 12.16. El pozo requiere estimulación y mayor
área perforada
87
Ploteo de Jones, Blount y Glaze

88
Prueba de 4 puntos
Problema 18): Datos: Pr = 1790 psia ( Pb > 1790 psia).
Una prueba dá q = 95 b/d a pwf= 1500 psia EF = 1
Calcular: (1) qomax a la presión estática de 1600 psia
(2) qo para Pr = 1600 psia y Pwf = 1000 psia.
Solución:(1) qomax a Pr = 1790 psia:
qo
qmax = ------------------------------------ = 351 b/d
1 -0.2(Pwf/Pr)-0.8(Pwf/Pr)²
(2) qmax a Pr = 1600 psia. Usando la relación:
qomax1 Pr1 1790
------ = ( ------- )3 = (------- )3 = 1.40
qomax2 Pr2 1600
qomax2 = 351/1.40 = 251 b/d.
(3) Usando la ecuación de Vogel para encontrar qo
para Pwf = 1000 psia y Pr = 1600 psia.
qo2 = qmax2[1 - 0.2(Pwf/Pr) - 0.8(Pwf/Pr)²] = 141 b/d
89
Pozo de Gas
Problema 19): Pozo de gas : Presión del reservorio
4500 psi Permeabilidad formación 40 md; radio de
drenaje 2100’; radio del hueco 0.45’ ;h de formación
60’; s = 0; gg = 0.65; Tres= 220F; hp= 40’.Construir la
curva IPR del pozo con y sin turbulencia. Utilizar
funciones p cuadrado y mp.
Solución:
1) Función p cuadrado
Pr2-Pwf2 = aq + bq2 ; donde:
1424Tmz 1424x680mz 2100
a= [pD+s] = (ln - 0.75)
kh 40x60 0.45
a = 3107mz
3.166x10-12b gg z T 2.33x1010
b= donde b =
hp² rw k1.201
90
Pozo de Gas
Pres Pavg mz a Qst z b Qct
psi psi MSCFD MSCFD
4500
4300 4401 .0226 70.20 25073 .969 0.000523 21599
4000 4257 .022 68.33 62196 .961 0.000518 46086
3000 3824 .0204 63.36 177550 .938 0.000506 99114
2000 3482 .0192 59.64 272490 .923 0.000498 130444
1000 3260 .0184 57.15 336831 .915 0.000494 147910
500 3202 .0183 56.84 351866 .913 0.000492 151918
0 3182 .0182 56.53 358222 .912 0.000492 153416
2) Funcion mp
La ecuación (7) puede ser escrita como:
m(Pr) - m(Pwf) = aq + bq2
1424T 1424x680 2100
a= [pD+s] = (ln 0.75)
kh 40x60 0.45
a = 3107

91
Pozo de Gas
3.166 x 10-12b gg T 2.33x1010
b= donde b =
hp² rw m k1.201
Pres uz 2p/uz 2p/uz Dmp mp Qst b Qct
psi E-2 prom. E8 E8 E-2
4500 2.3 391304 389346 0.78 11.3
4300 2.22 387387 384170 1.15 10.5 25071 2.3 21598
4000 2.1 380952 360931 3.61 9.38 62177 2.4 46074
3000 1.76 340909 302034 3.02 5.77 178381 2.5 99435
2000 1.52 263158 203008 2.03 2.75 275623 2.6 131427
1000 1.4 142857 107660 0.54 0.72 340983 2.7 149075
500 1.38 72464 36232 0.18 0.18 358314 2.7 153685
0 364146 2.7 155024
El rate Q está en MSCFD

92
IPR Pozo de Gas

93
Pozo de Gas
Problema 19a): Pozo de gas: Presión del reservorio
5200 psi k formación 45 md; espaciamiento= 320
acres; radio del hueco 10 3/4”; h de formación 30';s
=0 gg = 0.7;Tres= 210F; hp= 20'; rw=0.45';fT=2.441”;e=
0.0006";mg=0.022 cp.Construir la curva IPR del pozo
y el rate de flujo para THP= 1000 psi
Solución:
1) Función p cuadrado
Pr2-Pwf2 = aq + bq2; donde:
1424Tmz 1424x670mz 2106
a= [pD+s] = (ln - 0.75)
kh 45x30 0.45
a = 5442.7mz
3.166x10-12b gg z T 2.33x1010
b= donde b =
hp² rw k1.201
94
Pozo de Gas
Pres Pavg mz a z b Q
psi psi MSCFD
5200
5000 5101 .0275 147.19 1.00 .0001988 11.93
4500 4863 .0260 139.40 0.98 .0001913 33.40
4000 4639 .0247 134.53 0.97 .0001927 48.50
3000 4245 .0228 124.27 0.94 .0001873 70.53
2000 3940 .0215 116.70 0.92 .0001827 84.81
1000 3744 .0206 112.07 0.91 .0001805 93.01
500 3694 .0204 110.91 0.91 .0001800 95.03
0 3677 .0203 110.52 0.90 .0001800 95.69
Las presiones intake con THP= 1000 psi son:
Presión Rate Q Presión Rate Q
psi MMSCFD psi MMSCFD
2524 15 4583 30
3167 20 5358 35
3854 25
El rate es 30.05 MMSCFD
95
Rate Pozo de Gas

96
Pozo de Gas
Problema 20): Pozo de gas: Presión del reservorio =
5200 psi; k= 45 md; s= 0 ; re = 2100'; gg= 0.7; rw= =
0.45' T. reservorio = 210F h= 30'; hp= 10'
Construir la curva IPR usando pseudopresiones.
Solución:
1) Función p²
z de gráficos y mg de la correlación de Lee. La
siguiente Tabla se tiene q en MSCF/D. La ecuación
de influjo de gas:
Pr² - Pwf² = aq + bq2
a = (1424Tmz/kh) [pD +s]
a = 1424x670mz/45x30[ln2100/.45-.75] = 5440.52 mz
3.166x10-12b gg z T 2.33x1010
b= donde b =
hp² rw k1.201
97
Pozo de Gas
Pres Pavg mz a Qst z b Qct
psi psi E-4
5150 5175 .0274 122.0 4242 1.006 8 3458
5100 5150 .0273 121.5 8475 1.004 8 6062
5050 5126 .0272 121.1 12697 1.002 8 8236
4900 5052 .0268 119.3 25395 0.997 8 13425
4800 5004 .0266 118.4 33777 0.993 8 16227
4500 4863 .0258 114.9 59115 0.982 8 23040
4250 4749 .0253 112.6 79704 0.974 8 27546
4000 4639 .0247 110.0 100396 0.966 8 31427
3000 4245 .0228 101.5 177724 0.938 8 42850
2000 3940 .0215 95.7 240707 0.919 7 49989
1000 3744 .0206 91.7 283935 0.908 7 54047
500 3694 .0204 90.8 294977 0.906 7 55010
- 3677 .0203 90.4 299196 0.905 7 55348
2) Función m(p)
m(pr) - m(pwf) = a q + b q²
a = (1424T/kh) [pD +s]
a =1424x670/45x30[ln2100/.45-.75]= 5440.52 98
Pozo de Gas
b = b dividido entre mz de función Dp².
Pres uz 2p/uz 2p/uz Dmp mp Qst b Qct
psi E-1 prom. E8 E6 E-2
5200 .276 376812 376362 18.8 1391
5150 .274 375912 376148 18.8 1372 4227 2.9 3448
5100 .271 376384 376625 18.8 1353 8451 2.9 6049
5050 .268 376866 376894 56.5 1335 12681 2.9 8228
4900 .260 376923 376697 37.7 1278 25380 3.0 13419
4800 .255 376471 374957 112.5 1240 33841 3.0 16247
4500 .241 373444 372312 93.1 1128 59108 3.0 23038
4250 .229 371179 369921 92.5 1035 80015 3.1 27611
4000 .217 368664 353823 353.8 942 100788 3.1 31500
3000 .177 338983 304627 304.6 588 180263 3.3 43197
2000 .148 270270 209209 209.2 284 248687 3.4 50906
1000 .135 148148 111668 55.8 75 295680 3.5 55270
500 .133 75188 37594 18.8 19 308221 3.5 56358
0 312443 3.5 56684

99
Pozo de Gas-Isocronal Modificada
Ecuación: q = C (pws2-Pwf2)n
pws pwf q pws2-Pwf2 log(pws2-Pwf2) log q
1948 1784 4.5 0.61205 -0.2132 0.6532
1927 1680 5.6 0.89093 -0.0502 0.7482
1911 1546 6.85 1.26181 0.1010 0.8357
1887 1355 8.25 1.72474 0.2367 0.9165
extendida:
1948 1233 8 2.27442 0.3569 0.9031
q en mmscfd; p en psia
Pendiente = n = 0.5843
eje y = log q, eje x = log (pws2-Pwf2)
C = 8/(2.27442 x106) 0.5843 = 0.001544
AOF = 0.001544x (19482) 0.5843 =10.7892548 mmscfd

100
Pozo de Gas-Isocronal Modificada

101
Pozo de Gas

102
IPR Compuesto
Problema 22): Pr = 2250 psi. Pb = 3000 psi.
Una prueba arroja un rate total qt = 900 b/d para un
Pwf = 1800 psi.Encontrar y plotear la curva del IPR
compuesto para Fw =0, 0.25, 0.5, 0.75 y 1.00
Solución: A = 0.328
Fw J Qomax Pwfd Pwfc CD=Pd-Pc CG
0 2.195121 2743.902 11.0336 0 11.03357 2.743902
0.25 2.142857 2678.571 258.5876 250 8.58768 2.678571
0.5 2.093023 2616.279 506.1417 500 6.141786 2.616279
0.75 2.045454 2556.818 753.6958 750 3.695893 2.556818
1 2 2500 1001.25 1000 1.25 2.5
Tanb Tana Qtmax
4.0211 .2487 2743.902
3.2061 .3119 2756.548
2.3475 .4260 2829.269
1.4455 .6918 3075.667
0.5 2 4500
103
IPR Compuesto
Pwf A C D qt Fw=0
2250 9 .029155 0 0
2000 8.1111 .029155 -15.2098 521.680
1800 7.4 .029155 -26.24 899.999
1400 5.9778 .029155 -45.2661 1552.573
1000 4.5556 .029155 -60.2469 2066.393
800 3.8444 .029155 -66.2202 2271.271
400 2.4222 .029155 -75.1328 2576.962
0 1 .029155 -80 2743.9
B = 0 para Fw = 0
Pwf A C D qt Fw=0.25
2250 9 0.039835 0 0
2000 7.815 0.038522 -19.9286 515.20849
1800 6.867 0.037472 -33.8488 896.71663
1400 4.970 0.035372 -56.2954 1570.1421
1000 3.074 0.033271 -71.5500 2109.4580
800 2.126 0.032221 -76.4804 2322.2512
400 0.230 0.03012 -80.9472 2617.6268
0 - - - 2756.55
B = 5.53E-4 para Fw = 0.25 104
IPR Compuesto
Pwf A C D qt Fw=0.5
2250 9 0.0612 0 0
2000 7.222 0.0552 -28.840 509.307
1800 5.8 0.0503 -47.3688 895.225
1400 2.956 0.0406 -72.2654 1596.821
1000 0.111 0.0310 -80.9880 2174.255
800 -1.31 0.026121 -79.2814 2398.259
400 - - - 2658.833
B = 1.7E-3 para Fw = 0.5
Pwf A C D qt Fw=1 qt Fw=1
2250 9 0.125 0 505.35158 500
2000 5.444 0.088 51.358 898.12195 900
1800 2.6 0.058 -74.24 1640.5034 1700
1400 -3.089 -0.001 -71.459 2282.2097 2500
1000 -8.778 -0.060 -3.951 2518.5463 2900
800 -11.62 -0.090 54.07604 2798.1993 3700
400 -17.31 -0.149 218.6745 3076 4500
B = 5.2 E-3 para Fw = 0.75

105
IPR Compuesto

106
Curvas de Declinación
Problema 23): Demostrar que la primera
derivada de la razón de pérdida con respecto al
tiempo es la constante hiperbólica.
Definición de razón de pérdida:
a = -q/(dq/dt) = (1/k)q-n = (1/k)(qi-n+nkt)
a = (qi-n/k) + nt
da/dt = n
Problema 24): Demostrar a partir de la fórmula
hiperbólica la ecuación exponencial n=0.
qt = qi [(1+nDit)1/nDit]-(nDit)/n
lim(1+x)1/x x--->0 lim(1+x)1/x = e
qt = qi e-Dt
107
Curvas de Declinación
Problema 25): Fórmulas de aplicación hiperbólica a
partir del cociente r = qi / qt.
qi r1-n-1 Np (1-n) r - r1-n
Np = --------- [ ------- ]; t = ----- -------- [ --------- ]
(1-n)Di r1-n qi n r1-n-1
Datos: Np = 55900 bls; qi = 6292 bls/mes; t = 27 me-
ses; qt = 730 bls/mes. Encontrar n y Di.
Solución: r = 6292/730 = 8.62
De la ecuación de t:
55900 (1-n) 8.62 -8.621-n
27= --------- ------ [ ------------------ ]
6292 n 8.621-n - 1
iterando n = 0.6 ; 27 = 27;
Di = (rn - 1) / nt = 0.163

108
Futuras Curvas IPR
Problema Futuras Curvas IPR: Pruebas de Flujo
fueron tomadas sobre un pozo, cuando la Produc-
ción Acumulativa fué 18,400 bls y la Pr= 900 psi.
Rate b/d Pwf psi
85 650
120 520
130 480
La siguiente información adicional ha sido obtenida
durante la vida del pozo.
Pr psi J b/d/psi Np bls
1280 -
2.1 2,000
0.7 13,000
900 0.37 18,400
700 30,000
Las pruebas son a drawdown alrededor de 100 psi.
109
Futuras Curvas IPR

110
Futuras Curvas IPR
Se desea encontrar la siguiente información:
1)Indice de Productividad J a un límite económico
asumido de 40,000 bls de Producción Acumulada.
2)Drawdown necesario para producir a un rate al
límite económico asumido de 4 b/d.
3)Encontrar las presiones del reservorio para Np=
13000 y 40000 bls. Ploteando en semilog PR vs Np.
4)Al drawdown determinado en 2), calcular qo para
los diferentes Np.
5)Determinar la Ecuación q= C(Pr-Pwf)n para los
datos de la prueba de flujo y luego a las
Producciones Acumuladas de 13,000, 18,400,
30,000 y 40,000 bls.
6)Construir la curva IPR para la Producciones
Acumuladas de 13000, 18400, 30000 y 40000 bls.
111
Pr vs Np

112
Futuras Curvas IPR
Solución:
1) El Ploteo semilog de J en el eje log vs Np.
Del ploteo obtenemos que J= 0.037 b/d/psi para
Np = 40,000 bls.
2) El drawdown necesario para producir al límite
económico será 4/0.037 = 100 psi.
3) Ploteo en semilog de PR vs Np.
Pr psi J b/d/psi Np bls q bl/d
1280 -
1000 0.7 13,000 70
900 0.37 18,400 37
700 0.11 30,000 11
580 0.037 40,000 4
4) Donde qo ha sido obtenido utilizando los valores
dados de J para diferentes Np, a drawdown= 100 psi
113
Futuras Curvas IPR
5) El ploteo de la curva q=C(Pr-Pwf)n para Np= 18,400
bls Pr= 900 psi es:
Rate b/d Pwf psi (Pr-Pwf)
85 650 250
120 520 380
130 480 420
La ecuación es q= 0.809(Pr-Pwf)0.83
Calculamos C con valores de q para Dp= 100psi y
n=0.83
Pr psi Np bls q bl/d C
1000 13,000 70 1.531
900 18,400 37 0.809
700 30,000 11 0.241
580 40,000 4 0.088
114
Futuras Curvas IPR
Los IPR's son:
Presión q a Np
13000 18400 30000 40000
1000 0
900 70 0.00
700 174 65.74 0.00
580 230 97.10 12.82 0.00
400 310 140.64 27.42 6.55
300 352 163.61 34.81 9.45
200 393 185.94 41.90 12.18
100 434 207.74 48.74 14.79
0 473 229.07 55.39 17.30
Luego ploteamos el gráfico:

115
Futuras Curvas IPR

116
Futuras Curvas IPR

117
Futuras Curvas IPR
Problema Futuras Curvas IPR: Pruebas de Flujo
fueron tomadas en un pozo, cuando la Producción
Acumulativa fue 45,000 bls y la Pr= 1600 psi.
Rate b/d Pwf psi
200 1200
250 1000
300 750
La siguiente información adicional ha sido obtenida
durante la vida del pozo.
Pr psi J b/d/psi Np bls
2800 5.00
2200 1.8 20000
2000 1.00 30000
1600 0.46 45000
Las pruebas fueron tomadas a drawdowns de 300 a
400 psi
118
Futuras Curvas IPR
Se desea encontrar la siguiente información:
1) Indice de Productividad J a un límite económico
asumido de 104 bls de Producción Acumulada.
2) Drawdown necesario para producir a un rate al
límite económico asumido de 8 b/d.
3) Hallar las presiones del reservorio para Np=
80000 y 100000 bls, con ploteo semilog PR vs Np
4) Al drawdown determinado en 2), calcular qo para
los diferentes Np.
5) Determinar la Ecuación q= C(Pr-Pwf)n para los
datos de la prueba de flujo y luego a las Produc-
ciones Acumuladas de 45, 60, 80 y 100 x 103 bls.
6) Construir la curva IPR para las Producciones
Acumuladas de 45, 60, 80 y 100 x 103 bls.
Solución:
1) Ploteo semilog de J en el eje log vs Np. De allí
obtenemos que J= 0.025 b/d/psi para Np = 105 bls.
119
Futuras Curvas IPR

120
Futuras Curvas IPR
2) El drawdown necesario para producir al límite
económico será 8/0.025 = 320 psi.
3)Ploteo en semilog de PR vs Np.
Pr psi J b/d/psi Np bls q bl/d
2800 5.0 - 1600
2200 1.8 20,000 576
2000 1.0 30,000 320
1600 0.46 45,000 147.2
1360 0.21 60,000 67.2
1080 0.073 80,000 23.4
840 0.025 100,000 8
4) Donde qo ha sido obtenido utilizando los valores
dados de J para diferentes Np, a un drawdown de
320 psi.

121
Futuras Curvas IPR

122
Ploteo de Fetkovich
5) El ploteo de Fetkovich para Np= 45x103 bls,
Pr=1600 psi, es:
Rate b/d Pwf, psi 1-(Pwf/Pr)2
147 1280 0.360
200 1200 0.438
250 1000 0.609
300 750 0.780
La ecuación es:q = 390 (1-(Pwf/Pr)2)0.9
6) Calculamos qomax con q para Dp= 320 psi
Pr psi J b/d/psi Np bls q bl/d qmax bl/d
2800 5.0 - 1600 6368
2200 1.8 20,000 576 1873
2000 1.0 30,000 320 962
1600 0.46 45,000 147.2 369
1360 0.21 60,000 67.2 148
1080 0.073 80,000 23.4 43
840 0.025 100,000 8 12
123
Ploteo de Fetkovich

124
IPR - Fetkovich

125
IPR - Fetkovich
7) Los IPR’s son:
Presión Np
30000 45000 60000 80000 104
2000 0
1600 383.57 0
1360 0
1280 598.70 156 21.00
1200 643.78 185 38.11
1080 60.39 0
1000 742.56 250 73.48 7.45
840 96.05 18.64 0
750 839.34 312 106.79 23.78 2.85
500 907.71 356 129.87 34.61 8.09
350 935.44 373 139.15 38.91 10.1
200 953.34 385 145.12 41.67 11.4
100 959.84 389 147.28 42.67 11.9
0 962 390 148 43.00 12.0
Luego ploteamos el gráfico:
126
Futuras Curvas IPR

127
Flujo Multifásico Vertical
Problema 26): flujo multifásico vertical:
Método de Poettmann y Carpenter:
Datos: d=1.995" THP =500 psi T1 = 80F gg =0.65
gw= 1.07 API crudo= 22 qo= 400 b/d qw= 600 b/d
w=70 dinas/cm (asumido constante) GLR = 500
scf/stb go=0.922. Gradiente de temperatura fluyente
0.02 F/pie.
Calcular la presión de fondo fluyente a 7550'
Solución:
(1)Determinar la masa por stbo.
m =350x0.922 +.0764x1250x.65 +350x1.07x1.5=
m = 323+62+562 = 947 lbm/stbo
(2) El rate de flujo de masa total por día es:
w = (400)(947) = 378800 lbm/día.
(3) Se prepara la siguiente Tabla:
128
Flujo Multifásico Vertical
P bo z rL rg vsL vsg HLNS rm Dz Sz
--------------------------------------------------------------------------------
500 1.04 .904 62.52 2.17 3.04 5.72 .347 23.1
700 1.06 .891 62.18 2.82 3.06 4.30 .416 27.5 1015 1015
900 1.08 .882 61.86 3.44 3.09 3.44 .473 31.1 901 1916
1100 1.10 .877 61.57 4.03 3.11 2.87 .520 33.9 823 2739
1300 1.11 .875 61.28 4.59 3.13 2.47 .559 36.3 768 3507
1500 1.13 .877 61.02 5.12 3.15 2.17 .593 38.3 726 4233
1700 1.15 .880 60.76 5.61 3.17 1.93 .621 39.9 695 4928
1900 1.17 .885 60.52 6.06 3.19 1.75 .646 41.2 671 5599
2100 1.18 .893 60.28 6.49 3.21 1.60 .667 42.4 651 6250
2300 1.20 .901 60.06 6.87 3.23 1.48 .686 43.4 636 6886
2500 1.22 .912 59.85 7.22 3.25 1.38 .702 44.2 624 7510
2700 1.23 .923 59.64 7.55 3.27 1.30 .717 44.9 614 8124
Rs = 0.65 [ (P/18)x 1.5]1.2
F = Rs[0.65/0.922]0.5+1.25x110= 0.84 Rs + 137.5
bo = 0.972+1.47x10-4F1.175
Numerador Nº de Reynolds = d v rm
Numerador Nº Reynolds=1.4737x10-5 w/d = 33.6; f = 0.00802
(4) La presión a 7750' será 2578 psi.
129
Flujo Multifásico Vertical
Problema 26a): El mismo que el anterior pero utilizando el
método de Hagedorn y Brown.
PE2 bo z rL rg vsL vsg HL rm  Dz Sz
-----------------------------------------------------------------------------
5 1.04 .903 62.6 2.2 3.04 5.68 .559 35.9 .022 753 0
7 1.05 .886 62.3 2.9 3.06 4.22 .559 36.1 .022 750 753
9 1.07 .875 62.0 3.5 3.08 3.35 .571 36.9 .022 736 1503
11 1.09 .868 61.7 4.2 3.10 2.78 .586 37.9 .021 728 2239
13 1.11 .866 61.4 4.7 3.12 2.38 .607 39.1 .021 700 2958
15 1.13 .867 61.2 5.3 3.15 2.09 .621 40.0 .021 686 3658
17 1.14 .870 60.9 5.8 3.17 1.87 .632 40.6 .021 677 4344
19 1.16 .876 60.6 6.2 3.19 1.69 .654 41.8 .021 659 5021
21 1.18 .884 60.4 6.7 3.21 1.55 .675 42.9 .021 643 5680
23 1.20 .894 60.2 7.0 3.23 1.43 .693 43.8 .021 631 6323
25 1.21 .904 60.0 7.4 3.25 1.33 .709 44.6 .021 620 6954
27 1.23 .916 59.7 7.7 3.27 1.25 .723 45.3 .021 612 7574
29 1.25 .928 59.5 8.0 3.29 1.18 .736 45.9 .021 605 8186

130
Flujo Multifásico Vertical
P NvL Nvg CNL ND ø NL
---------------------------------------------------------------
500 6.23 11.61 .0035 22.49 .00035 .035
700 6.29 8.68 .0029 22.64 .00035 .022
900 6.36 6.92 .0026 22.80 .00036 .015
1100 6.42 5.76 .0024 22.95 .00039 .011
1300 6.49 4.95 .0024 23.08 .00042 .008
1500 6.55 4.35 .0023 23.18 .00044 .006
1700 6.60 3.89 .0022 23.24 .00046 .005
1900 6.65 3.52 .0021 23.29 .00048 .004
2100 6.71 3.23 .0021 23.37 .00050 .003
2300 6.76 3.00 .0020 23.45 .00051 .003
2500 6.81 2.80 .0020 23.49 .00053 .002
2700 6.85 2.63 .0019 23.50 .00054 .002
2900 6.89 2.48 .0019 23.46 .00056 .002
La presión a 7750' será 2750 psi.

131
Flujo Multifásico Vertical
Problema 26 b) El mismo que el anterior pero utilizando el
método de Beggs y Brill.
PE2 bo z rL rg vsL vsg HL rm  Dz Sz
-------------------------------------------------------------------------------
5 1.04 .903 62.6 2.2 3.04 5.68 .478 31.0 .052 728 0
7 1.05 .885 62.3 2.9 3.06 4.21 .531 34.4 .042 717 728
9 1.07 .873 62.0 3.5 3.08 3.34 .571 36.9 .037 697 1445
11 1.09 .866 61.7 4.2 3.10 2.76 .602 38.8 .033 678 2142
13 1.11 .863 61.4 4.8 3.12 2.36 .626 40.2 .031 662 2819
15 1.13 .863 61.2 5.3 3.14 2.06 .646 41.4 .030 650 3482
17 1.14 .866 60.9 5.9 3.17 1.84 .662 42.3 .028 640 4131
19 1.16 .871 60.7 6.3 3.19 1.66 .675 43.0 .028 633 4771
21 1.18 .879 60.4 6.8 3.21 1.52 .686 44.1 .027 620 5403
23 1.20 .889 60.2 7.2 3.23 1.40 .696 45.0 .026 609 6023
25 1.21 .899 60.0 7.5 3.25 1.31 .704 45.8 .026 600 6632
27 1.23 .911 59.8 7.9 3.27 1.23 .711 46.4 .025 593 7232
29 1.24 .923 59.6 8.2 3.29 1.16 .717 47.0 .025 587 7825
La presión a 7750' será 2875 psi.
132
Flujo Multifásico Vertical

133
Flujo Multifásico Vertical
Problema 27: Método de Poettmann y Carpenter
Datos: d = 1.995" TH1 = 500 psi T1 = 120F THP2 = 1000 psi 
Temp 0.02 F/pie; e=.00015’;gg = 0.65 gw= 1.07API crudo= 22 qo=
400 b/d qw= 600 b/d w= 70 dinas/cm (asumido constante) GLR
= 500 scf/stb go=0.922 GOR=[(600+400)/400]x500= 1250 scf/stb.
Calcular distancia entre puntos de presión 500 y 1000 psi
Solución:(1)Determinar la masa por stbL.
M=go(350)[(1/(1+WOR)]+(350)[(WOR/(1+WOR)]+(0.0764)(GLR)gg
M= 129 + 224.5 + 24.8 = 378.5854 lbm/stbL
(2) Calcular el rate de flujo de masa w
w = (378.585)(1000) = 378,585.4 lbm/d.
P bo z rL rg vsL vsg HLNS rm Dz Sz
----------------------------------------------------------------------------------
500 1.05 .929 62.15 1.86 3.06 6.74 .312 20.66 677
625 1.07 .922 61.93 2.23 3.07 5.55 .356 23.50 627 677
750 1.08 .916 61.72 2.59 3.09 4.72 .396 25.98 587 1305
875 1.09 .912 61.53 2.94 3.10 4.11 .430 28.15 554 1891
1000 1.10 .909 61.34 3.28 3.12 3.63 .462 30.08 527 2445
=Factor de fricción= 0.00802. La distancia será 2445’ 134
Flujo Multifásico Vertical
Problema 27 a): El mismo que el anterior pero por el Método de
Hagedorn y Brown:
Solución:
Presión bo z rL rg vs l vsg HL ƒ rm Dz Sz
500 1.05 .928 62.16 1.86 3.06 6.72 .4820 .02091 30.927 537.3 0
625 1.06 .920 61.97 2.25 3.07 5.50 .4965 .02086 31.902 523.4 537
750 1.07 .913 61.78 2.62 3.08 4.65 .5107 .02082 32.837 510.6 1061
875 1.09 .908 61.60 2.99 3.10 4.03 .5269 .02080 33.871 497.1 1571
1000 1.10 .904 61.42 3.34 3.11 3.56 .5426 .02078 34.855 484.9 2068

Presión NVL NVG CNL ND  FI XNL

500 6.25 13.74 .00277 22.37 1.00 .00025 .019


625 6.29 11.27 .00258 22.48 1.00 .00026 .015
750 6.33 9.56 .00246 22.57 1.00 .00028 .011
875 6.37 8.30 .00239 22.66 1.00 .00030 .009
1000 6.42 7.35 .00235 22.76 1.00 .00032 .008

135
Flujo Multifásico Vertical
Problema 27 a): El mismo que el anterior pero por el Método de
Hagedorn y Brown:
Solución:(1)Determinar la masa por stbL.
m= go(350)[(1/(1+WOR)]+(350)[(WOR/(1+WOR)]+(0.0764)(GLR)gg
m = 129+224.5+24.8 = 378.5854 lbm/stbL
(2)Calcular el rate de flujo de masa w
w = (378.585)(1000) = 378,585.4 lbm/d.
Primera iteración Dz= 2000':
T2 = 160F
Rs1 = 58.15 scf/stb bo1= 1.05 rb/stb z1= .933
mom1= 28.96 cp mo1= 16.08 mw1= .615 cp
Rs2 = 119.09 scf/stb bo2= 1.09 rb/stb z2= .905
mom2= 10.39 cp mo2= 4.56 cp mw2= .425 cp
moavg = (16.08+ 4.56)/2 = 10.32 cp
mwavg = (.615+.425)/2 = .520 cp
mL = 10.32(400/1000)+.520(600/1000)= 4.44 cp
136
Flujo Multifásico Vertical
Pavg = (500+1000)/2+14.7 = 764.7 psia
Tavg = (T1+T2)/2 = 140F
Rsavg= 58.15 scf/stb boavg= 1.05 rb/stb zavg= .933
rL=((.922x62.4+ 58.15x.65x.0764/5.615)/1.05)x(400/1000) +
1.07x62.4x(600/1000) = 61.89 lb/pie3
rg = 2.7x764.7x0.65/(600x.933) = 2.44 lb/pie3
om = 32.34 dinas/cm o= 18.12 dinas/cm
L = 18.12x(400/1000) + 70x(600/1000) = 49.25 dinas/cm
NL = .15726x4.44x(61.89x49.253)-.25 = 0.0134
CNL =0.00254
AT = 3.1416x.16625²/4 = 0.022 pies²
vsL= (400x1.05+600)x5.615/(86400x0.022)= 3.076 pies/seg
NvL= 1.938x3.076x(61.89/49.25).25 = 6.31
bg = .0283x600x.933/764.7 = .0207 cf/scf
vsg= 400x(1250-58.15)x.0207/(86400x0.022)= 5.03
pies/seg
Nvg= 10.32
137
Flujo Multifásico Vertical
ND = 120.872x.16625x(61.89/49.25)½ = 22.53
f = 6.31x(764.7/14.7).1x0.00254/(22.53x10.32.575)
f = .000276
HL/¥ = .5038
F = .00255 ¥ = 1
HL = .5038
rm = 61.89x.5038+2.44x.4962 = 32.395 lb/pie3
mg = 0.01322 cp
Nre2=
2.2x10-2x378585/(.16625x4.44.5038x.01322.4962)=202234
e/d = .000902  = .0209
vsL1= 3.048 lb/pie3 vsg1= 7.78 lb/pie3
vm1 =10.83 lb/pie3
vsL2= 3.104 lb/pie3 vsg2= 3.83 lb/pie3
vm1 = 6.93 lb/pie3
D(vm)² = 69.24
138
Flujo Multifásico Vertical
Dz =
(144x500-32.395x(69.24/64.34))/(32.395+.0209x378585²/
(2.9652x1011x32.395x.166255)) = 2066.213 pies
Segunda iteración:
Dz= 2066.213'
T2 = 161.32F
Rs1 = 58.15 scf/stb bo1= 1.05 rb/stb z1= .933
mom1= 28.96 cp mo1= 16.08 mw1= .615 cp
Rs2 = 118.70 scf/stb bo2= 1.09 rb/stb z2= .906
mom2= 10.13 cp mo2= 4.48 cp mw2= .42 cp
moavg = (16.08+4.48)/2 = 10.28 cp
mwavg = (.615+.42)/2 = .518 cp
mL = 10.28(400/1000)+.518(600/1000)= 4.42 cp
Pavg=(500+1000)/2+14.7=764.7 psia
Tavg=(T1+T2)/2 = 140.66F
Rsavg= 88.94 scf/stb boavg= 1.07 rb/stb zavg= .916
139
Flujo Multifásico Vertical
rL=((.922x62.4+ 88.94x.65x.0764/5.615)/1.07)x(400/1000)
+ 1.07x62.4x(600/1000) = 61.88 lb/pie3
rg = 2.7x764.7x0.65/(600x.916) = 2.44 lb/pie3
om = 32.34 dinas/cm o= 18.12 dinas/cm
L = 18.12x(400/1000) + 70x(600/1000) = 49.25 dinas/cm
NL = .15726x4.42x(61.88x49.253)-.25 =0.0133
CNL =0.00254
AT = 3.1416x.16625²/4 = 0.022 pies²
vsL= (400x1.07+600)x5.615/(86400x0.022)= 3.076 pies/seg
NvL= 1.938x3.076x(61.88/49.25).25 = 6.31
bg = .0283x600x.916/764.7 = .0203 cf/scf
vsg= 400x(1250-88.94)x.0203/(86400x0.022)= 5.03 pies/seg
Nvg= 10.33
ND = 120.872x.16625x(61.89/49.25)½ = 22.53
f = 6.31x(764.7/14.7).1x0.00254/(22.53x10.33.575)
f = .000276
HL/¥ = .5035 140
Flujo Multifásico Vertical
F = .00255 ¥ = 1
HL = .5035
rm = 61.88x.5035+2.44x.4965 = 32.368 lb/pie3

mg = 0.01322 cp
Nre2=
2.2x10-2x378585/(.16625x4.42.5038x.01322.4962)=202997
e/d = .000902  = .0209

vsL1= 3.048 lb/pie3 vsg1= 7.78 lb/pie3


vm1 =10.83 lb/pie3
vsL2= 3.104 lb/pie3 vsg2= 3.84 lb/pie3
vm2 = 6.94 lb/pie3
D(vm)² = 69.05
Dz =
(144x500-32.368x(69.05/64.34))/(32.368+.0209x378585²/
(2.9652x1011x32.368x.166255)) = 2067.701 pies
Se puede hacer más iteraciones.
141
Flujo Multifásico Vertical
Problema 27 a): A través de otro procedimiento:
Ecuación gradiente en un punto de la tubería vertical:
dp  rf vm2
--- = rm senø + ---------
dh 2 gc d
donde rf = rn2 / rm
rn = rLHLNS + rg(1-HLNS)
HLNS = vsL/vm
Para la 1ra. iteración:
vsL = 3.037 pies/seg; vm = 3.037 + 4.96 = 8.00 pies/seg
HLNS= 3.037/8.00 = 0.3828
rL = 61.88 lb/pie3
rg = 2.44 lb/pie3
rn = 61.88 x 0.3828 + 2.44 x (1-0.3828) = 25.19 lb/pie3
rf = 25.192/32.37 = 19.608 lb/pie3
mg = 0.01322 cp
Nre2= 202,234
e/d = .000902  = .0209
dp .0209x19.0628x8.002
--- = 32.37 + --------------------------- = 34.82
dh 2x32.2x.16625
Dz = 144x(500/34.82)= 2,068'
142
Flujo Multifásico Vertical
Problema 27 b): El mismo problema que el 27), pero por el
método de Orkiszewski.
1) Determinar el patrón de flujo
a) número de la velocidad de gas
Nvg 10.19 pies/seg  vg
b) límites de frontera
(L)B = 1.071 - (0.2218 vt2/d)
vt = vsg+vsL = 4.96+3.04= 8 pies/seg
(L)B= 1.07 - (0.2218 64/0.16625) = -84.31
qg/qt = vsg/vt = 4.96/8 = 0.616
0.616 > -84.31 no es burbuja
(L)S = 50+(36 vgx qL/qg) = 50+36(10.19x0.613) = 274.87
qL/qG = vsL/vsg= 3.04/4.96 = 0.613
0.613 > -84.31 , 10.19< 273.06
patrón de flujo slug
2) Cálculo densidad promedio y gradiente por fricción.
a) NRe = 1488 rLvtd/mL = 27706.56
rL = 61.88 lb/pie3
mL = 4.42 cp
b) Cálculo del Número de Reynolds de burbuja
Nb = (1488)(61.88)(0.16625) vs / 4.42
asumiendo vs = 2.03
Nb = 7030.5 143
Flujo Multifásico Vertical
como 3000< Nb < 8000
vsi=(0.251+8.74E-06 NRe)(32.2x0.16625)½
vsi= 1.141
vs=½[1.141+((1.141)²+(13.59)(4.42)/(61.88x(0.16625)½)½
vs = 1.53
Nb = 5298.97
c) Cálculo coeficiente de distribución liquida  y f.
=[(0.013)(log4.42)/(0.166251.38)]-0.681+0.232log8- -0.428log0.1
6625
=0.0998-0.681+0.2095+0.333= -3.82E-02
  -0.065x8  -0.52
e/d = 9E-04 f = 0.025
d)Evaluación de la densidad promedio fluyente
wt = 378300 lbm/día=378300/86400=4.378 lbm/seg

_ wt+rLvsAp
r = -------------- +rL
qt+ vs Ap

144
Flujo Multifásico Vertical
_ 4.378+61.88x1.53x.0217
r= - 3.28E-02x61.88
0.1736 + 1.53x0.0217
Ap = 0.0217 pies² ; qt=vtAp=8x0.0217=0.1736 pies/seg
_
r = 28.74 lb/pie3
e)Evaluación pérdida de fricción por pared
fLvt2rL qL+vsAp
f = ---------- [------------ +  ] = 4.083
2gcd qt+vsAp
qL= vsL Ap = 3.04x0.0217 = 0.066 pies3/seg
3) Determinación de Dh
4.378x0.107
(500)(144)[1- ]
4637x0.0217²x764.7
Dh = = 2193'
28.74 + 4.083
145
Flujo Multifásico Vertical
Problema 27 c): El 27) por el método de Beggs y Brill.
Presión bo z rL rg vs l vsg HL ƒ rm Dz Sz
500 1.05 .928 62.17 1.87 3.06 6.72 .4487 .03395 28.924 510.9 0
625 1.06 .919 61.98 2.25 3.07 5.49 .4850 .03232 31.221 493.5 511
750 1.07 .912 61.80 2.63 3.08 4.64 .5152 .03103 33.114 477.9 1004
875 1.09 .907 61.62 3.00 3.10 4.01 .5407 .02998 34.695 464.6 1482
1000 1.10 .902 61.45 3.36 3.11 3.54 .5624 .02911 36.030 453.3 1947

146
Flujo Multifásico Vertical
Problema 27 c): El 27) por el método de Beggs y Brill.
1) Colgamiento sin deslizamiento
L = qL/(qL+qg)= vsL/(vsL+vsg)=3.04/(3.04+4.96)=0.38
2) Número de Froude, viscosidad liquida, tensión superficial
liquida y viscosidad de la mezcla.
NFR= vm2/gcd = 8²/(32.17x0.16625)= 11.97
mL = 4.42 cp ;mg = Corr.de Lee = 0.01322 cp
mm=(6.27E-04)(4.42x0.38+0.01322x0.616)=0.001058lbm/ft-seg
L= 49.25 dinas/cm
3)Cálculos de los rates de flujo de masa
GL = rL vsL = 61.88x3.04 = 188.11 lbm/ft²-seg
Gg = rg vsg = 2.47x4.96 = 12.25 lbm/ft²-seg
Gm = GL + Gg= 188.11+12.25= 200.4 lbm/ft²-seg
4) Número de Reynolds sin deslizamiento y el Número de
velocidad liquida
NRes = Gm d/mm = (202.36x0.16625)/0.001058= 31483.8
NvL = 6.23
Calcular L1 , L2 , L3 y L4.
L1 = 316(£L)0.302 = 235.93
L2 = 0.0009252(£L)-2.4684 = 0.01
L3 = 0.1(£L)-1.4516 = 0.407
L4 = 0.5(£L)-6.738 = 339.15
147
Flujo Multifásico Vertical
6) Determinar el patrón de flujo
0.01  £ < 0.4 L3 < NFR  L1
Flujo intermitente.
7) Calcular el holdup horizontal
0.845 £0.531
HL(o) = = 0.485
NFR0.0173
8)Coeficiente de corrección por inclinación:
C=(1-£)ln[(2.96)£0.305NvL-0.4473NFR0.0978]
C=(1-0.38)(0.22) = 0.133
9)Factor de corrección por inclinación del
holdup líquido: ¥ = 1 +0.3C = 1.04
10) Holdup líquido y la densidad de dos fases
HL(90) = HL(o)¥ = 0.485x1.04 = 0.504
rtp = 61.88x0.504+2.47(1-0.504)= 32.4 lb/pie3
11) Calcular la razón del factor de fricción
£ 0.38
y=[ ]=[ ] = 1.496
HL902 0.504²
148
Flujo Multifásico Vertical
ftp lny
= exp[ ]
fns (-0.0523+3.182lny-0.8725[lny]²+0.01853[lny] } 4
(ftp/fns) = exp(0.37156) = 1.45
12) Calcular el factor de fricción sin deslizamiento
Nre = 31483.8
1
fns = = 0.022
[2log(NRens/(4.5223logNRens-3.8215))]²
13) Calcular el factor de fricción de dos fases
ftp
ftp = fns( ) = 0.022 (1.45) = 0.0319
fns
14) Calcular Dz
rtpvm vsg 32.5x8x4.96
144Dp [1 - ] 144x(500)[1- ]
gc p 144 32.17x764.7x144
Dz= =
ftp Gm vm 0.0319x200.36x8
tp + 32.4 +
2 gcd 2x32.17x0.16625
Dz = 1936'
149
Flujo Multifásico Vertical
Problema 27d):El mismo problema que el 27) pero por el
método de Duns y Ros:
1)Nvg =10.29
2) NvL = 6.23
3)Seleccionar el patrón de flujo. Fig.2.63
Región II
4)Determinar el factor de deslizamiento:
NL 0.0133
Nvg 10.19
Nd 22.53
de la Fig. 2.65 F5 = 0.19
F6 = -0.1
F7 = 0.085
'6
F = 0.029 Nd + F6 = 0.553
(1+F5)Nvg0.982 + F'6
s= = 3.5
(1+F7Nvg)²
5)Cálculo de la velocidad de deslizamiento
Región II
s 3.5
vs = = = 1.75 pies/seg
¼
1.938(rL/L) 1.938(61.88/54) ¼
150
Flujo Multifásico Vertical
L 54 dinas/cm
rL 61.88 lb/ft3
6)Cálculo del holdup
vs - vsg - vsL +[(vs - vsg - vsL)² + 4 vs vsL]½
HL =
2 vs
vs 1.75 pies/seg; vsg 4.96 pies/seg
vsL 3.04 pies/seg
HL = 0.4336
vsg vsL 4.96 3.04
vs = = = 1.75 pies/seg
1-HL HL 0.5664 0.4336
7) Cálculo del Número de Reynolds líquido:
(NRE)L = (1488 rL vsL d)/mL
rL = 61.88 lb/ft3
vsL= 3.04 pies/seg
d = 0.166 pies
mL = 4.42 cp
(NRE)L = 10.513x103
8) Determinar la gradiente por fricción adimensional
Región II
f1 de la Fig.2.67 f1 = 0.0092
f2 de la Fig.2.68 151
Flujo Multifásico Vertical
se entra con la abcisa
vsg
f1 ( ) Nd2/3 = 0.0092(4.96/3.04)(21.7)2/3 = 0.222
vsL
f2 = 1.01
f3 = 1+0.015 [(vsg/vsL)/50]½ = 1.003
fw = (f1)(f2/f3) = 0.015 x(1.01/1) = 0.0151
Gfr = 2 fw [(NvL+Nvg)(NvL)]/Nd = 0.137
9) Gradiente estática adimensional
Gst = HL + (1-HL)(rg/rL) = 0.456
rg ---> 2.44 lb/ft3
10)Gradiente total adimensional(fracción de la
gradiente estática liquida).
G= Gst + Gfr = 0.456+0.137 = 0.593
11)Gradiente Total
dp/dh = G rL = 0.593x(61.88/144) = 0.2548 psi/pie.
12)Distancia Dh
Dh = (500/0.2548) = 1962'
152
Cambio de Tubing
Problema 28):Uso curvas de Hagedorn y Brown. Profundidad
del pozo= 5200'; casing 7" @ 5050' PR = 1850 psig GLR = 0.4
mscf/stb. Tubing 2½" ø @ 5000' qo = 250 b/d CHP= 1245 psi.
El tubing se corroe y debe ser reemplazado, evaluándose
tubos de 2" , 2½" y 3½".
Que diámetro de tubería podría correrse, si se desea que el
pozo fluya al rate máximo posible con THP = 170 psig.
Solución:
pwf = 1260[1+(51.5/100)]=1400 psia = 1387 psi
(1) De la ecuación de Vogel
qmax = qo/1 - 0.2(Pwf/Pr) - 0.8(Pwf/Pr)² = 624 b/d
(2)Determinación del IPR. De la ecuación de Vogel:
Pwf= 0.125 Pr[-1+ 81-80(qo/qomax)]
qo Pwf THP 2" THP 2½" THP 3½"
0 1850.00
50 1765.95 310 250
100 1678.20 435 300 160
200 1489.33 430 400
400 1029.39 240 250
600 719.33 40 110
Para un THP= 170 psig se escoge tubería de 2½". 153
Cambio de Tubing

154
Evaluación de Tratamiento
Problema 29): Pozo que produce de una caliza:PR=
1700 psi Profundidad = 4052', casing 7" @ 4020';
tubos 2" a 4000';q = 360 b/d de petróleo limpio; GOR
= 600 scf/stb, THP= 300 psi. Se decide hacer acidifi-
cación, después del tratamiento se tiene q = 410 b/d,
tubos de 2" GOR = 1000 scf/stb y THP = 300 psi.
Determinar si el tratamiento es o no exitoso.
Rate a THP = 300 psi, si solo se reemplaza la sarta
de 2" por una de 3½".
Solución: Cálculos de pwf
qo Pwf Pwf d.t Pwf a.t Pwf 3½"
100 1501 630 730 1400
200 1275 580 770 820
400 664 600 830 700
500 0 620 880 700
Con cambiar tubería, se obtiene la misma produc-
ción que con la acidificación y con un menor GOR.
155
Evaluación de Tratamiento

156
Rate y Bean a un THP
Problema 30):Pozo con Ps= 2000 psi, completado
con tubos de 2"ø @ 5000'.El pozo produce 600 b/d,
THP = 200 psi y GOR 300 scf/stb y corte de agua del
10%. Cuál es el bean y rate con un THP de 100 psi.
Solución: Pwf para THP= 200 psi y 600 b/d de rate.
GLR= GOR ([(1/(1+WOR)] = 300x0.9 = 270 scf/stb.
Pwf =1360 psi. qomax= 1214 b/d
Se tiene la siguiente Tabla para THP= 100 psi
qo Pwf IPR Pwf 2"ø qo Pwf IPR Pwf 2"ø
100 1907 960 600 1360 1000
200 1809 920 800 1080 1120
400 1598 980 1000 722 1200
El rate será aproximadamente 780 b/d.
El bean para flujo crítico:
435 GLR0.546 q
S1.89 = = 50.55; Bean de 50.55/64"
THP 157
Rate y Bean a un THP

158
Permeabilidad pseudorelatina
Problema 31) Dados los siguientes datos para varias
zonas de la formación Mogollón, preparar la curva
de permeabilidad pseudo-relativa estática, asu-
miendo avance de agua de fondo desde la base de
la arena inferior.
Profundidad- Permeabilidad Porosidad Swi
pies md f

3,998-4,007 564 0.272 0.15


4,007-4,019 166 0.208 0.21
4,019-4,031 591 0.273 0.15
4,031-4,040 10.2 0.142 0.32
4,040-4,049 72 0.191 0.25
En todos los casos Sor = 0.3, k'ro a Swi es 0.9, k'rw a
Sor es 0.5.
159
Determinación de presiones
Problema 32): Un pozo ha estado fluyendo por 10
días a un rate de 200 STB/D. Propiedades de la roca
y fluido son: bo=1.13RB/STB; pi= 3000 psia; m= 0.5
cp; k=25 md.(uniforme al hueco -es decir, s=0); h =
50'; ct = 2x10-5 psi-1; f=0.16; y rw = 0.333'.
Calcular las presiones a 0.1,1 y 10 días, y plotear pwf
vs log t.
Problema 33):Determinar la pwf en un pozo en el
centro de un cuadrado, cuyos lados miden, L=250',
debido a la producción q=200 STBO/D. Propiedades
de la roca y fluido son: k= 50 md. s= 5; h=50'; ct=
2x10-5 psi-1; f=0.16; rw= 0.333'; m= 0.5 c; b =1.13
RB/STB.
El pozo está en pseudoestable, con una presión ps
=3,000psia.
160
Curvas de Declinación
Problema 34): La producción de un campo
petrolífero es de 23,000 b/d de petróleo. Si es
estima que sin perforación adicional la producción
dentro de una año será de 17,000 b/d de petróleo.
Cual será la reserva de petróleo si el límite
económico es de 2,500 b/d y la declinación es
hiperbólica con n=0.5 durante los cuatro primeros
años y con n=0.4 los restantes.
Problema 35): El historial de producción de un pozo
es el siguiente:
Barriles
Años 1 2 3 4
1er. Semestre 16,227 10,751 7,732 5,784
2o. Semestre 13,041 9,042 6,701 5,199
Cual será la reserva si el límite económico es de
1,000 barriles semestrales.
161
Problemas
Problema 36): Para reservorio de petróleo:gg = 0.65
API crudo =35; separador:T=80F, p =40 psia GOR
=1123 pies3/bl;Tres.=153F. Graficar presión vs Factor
de Volumen. 4 puntos saturado, 3 en bajo saturado.
Problema 37):Si se produce condensado de 50API
con un gas de gravedad 0.65, a una razón de 50
bl/MMcf, cuál es el factor de volumen a condiciones
del reservorio. Pres.= 3000 psia Tres.=220F
Problema 38) Dos pozos han sido perforados a una
distancia de una milla y se apartan justamente de la
trampa de petróleo. Debajo de la trampa de petróleo
inclinada, se tiene agua de fondo moviéndose a un
rate de 285 B/D. El espesor del acuífero es de 65', el
ancho 3000',m=1 cp y k=7.5 md. Asumiendo flujo
lineal y las gradientes del agua y petróleo de 0.45 y
0.35 psi/pie, respectivamente. Hallar la pendiente de
la interfase agua-petróleo.
162
Rate sin conificación
Solución:
7.5x3000x65
285=1.127x10-3 Dp
1x5280
D p =912.97 psi ; D piez = 912.97/0.45=2028.82'
0.45 x 2018.82
tg q = = 0.2161
0.80 x 5280
Problema 39): Un pozo ha sido totalmente baleado.
Dados: h = 140'; hcg = 50' ; b = intervalo perforado =
30‘; Espaciamiento : 80 acres. Kh= 90 md . kv=300
md. A condiciones del reservorio: go= 0.741; gw=
1.092 gg = 0.098; mo = 1.11 cp; bo = 1.25 RB/STB.
Empuje por capa de gas y agua de fondo, están
presentes, pero un activo empuje de agua no es
evidente de la historia del reservorio.
Hallar máximo rate de petróleo libre de gas y agua.
163
Rate sin conificación
Solución: re = (43,560x40)½
1320
rDe = 90/300 = 5.16
140
hcg / h = 50/140 = 0.357; hcw / h = 60/140 = 0.429
fb = 30/140 = 0.214 ¥g = 0.058 ¥w= 0.072
Dgog = 0.643 Dgow =0.351
qoc,g = 145.8 SRB/D qoc,w = 98.7 STB/D
El qoc es el menor 98.7 STB/D.
1320 Dgog
rDe = ------  90/300 = 5.16 ------ = 1.83
140 Dgow
hc fb ¥ hcg/h hcg qc
20' 0.1 0.055 0.355 71' 282
40' 0.2 0.053 0.325 65' 272
60‘ 0.3 0.049 0.27 54‘ 251
20' será la máxima capacidad sin conificación. 164
Rate sin conificación
Problema 40): Supongamos que en el campo
del ejemplo anterior, un pozo nuevo es
perforado. El espesor de la arena neta en el
pozo es 200'.
Determinar la longitud y posición del
intervalo perforado que permita el máximo
rate de petróleo libre de gas y agua. Probar
con 20,40 y 60' de intervalo perforado.
Si el pozo con 20,40 y 60' de intervalo
perforado es capaz de producir 850,1000 y
1100 B/D, respectivamente. Cuál sería el
intervalo perforado escogido.
165
Rate estable y f del bean
Problema 41): Pozo de gas y condensado de 8,000'
equipado con tubing de 1.995" ID y 5,280' de línea de
flujo de 2.441" ID. La presión en el separador es fija
de 800 psia. Cuál es el máximo rate estable y el
diámetro del bean, con los siguientes datos:
sp.gr gas = 0.68 Tr= 260F Ttf= 100F Tsep= 70F
mg= 0.012 cp e = 0.0018" n= 0.812 z = 0.95 C=
0.0345 MSCFD/psia1.624 Pr= 1800 psia API
condensado= 58 Rate Líquido = 80 STB/MMSCF
1.- Performance inflow.
S= 0.432039
Rates pwf Nre f ptf 0.7 ptf
psia psia
1000 1710.99 731897.7 .01968 1358.834 951.1840
2000 1582.86 1463795. .01943 1189.052 832.3368
3000 1424.68 2195693. .019341 917.293 642.1051
4000 1230.92 2927590. .019294 372.877 261.0141 166
Rate estable y f del bean
Performance outflow
Rates Nre f ptf
psia
1000 598171.2 0.019787 808.4942
2000 1196342. 0.019490 832.9660
3000 1794513. 0.019382 872.0403
4000 2392684. 0.019326 923.8606
Los rates están en MSCFD.
La intersección es en 2,000 MSCF/D
Estrangulador f = 0.35"

167
Rate estable y f del bean

168
Rates de producción
Problema 42): Pozo de petróleo, psep= 40 psi sp.gr
gas = 0.65 línea de flujo de 3,000' y 1.995" ID
API = 35 WOR= 0 T= 140F.Profundidad a la mitad
de las perforaciones = 5,200' tubing = 1.995" ID
GOR = 400 SCF/STB Pr = 2300 psi Pb= 1800 psi. El
pozo produce 800 STBO/D con pwf= 1500 psi.
a) Rate con bean de ¼"
b) Máximo rate estable
Rate con bean de ¼" = 400 STB/D
Rate máximo estable = > 1000 STB/D, pero no
cumple condición que el drawdown Dp< 50% de Pr.
Problema 43): Un campo tiene una producción de
16,000 B/D. Si la producción al límite económico es
3,000 B/D en 14 años:a) Calcular las Reservas
Probadas Desarrolladas bajo declinación exponen-
cial, hiperbólica n=0.5 y armónica.b) Plotear en
línea recta gráfico apropiado: el rate de producción
vs. tiempo, para los tipos de declinación anotados.
169
Rates de producción
Problema 44): Reservorio bajo saturado de la forma
debajo, con impulsión de agua en A (presión 3000
psia). La impulsión de agua es tal que el reservorio
se encuentra bajo flujo estable. Si la presión en el
pozo B es 2500 psia. Presión burbuja 2000 psia, mo=
0.84 cp, bo = 1.15 rb/stb, rw = 1/3', ko = 500md, Dpskin=
80 psi, go = 0.82. Cuál es el rate del pozo.
Solución: DPFLUJO = DPTOTAL - DPESTATICO - DPSKIN
DPFLUJO = 500-100x0.433x0.82- 80 = 384.494 psia
Entre el pozo y el contacto agua-petróleo hay una
parte de flujo radial y otra lineal.
Radio de drenaje para el flujo radial= 660/2 = 330'
px330²/2= L x 660; L = 259' a descontar al flujo lineal
0.84xqx1.15x(1320-259) 141.2x0.84xqx1.15xln990
384.494= +
1.127x660x10x0.5 500x10
384.494 = 0.276 q + 0.188 q
q = 828.35 STB/D.
170
Rates de producción
Problema 45): Un pozo ha sido totalmente baleado.
Datos: h=140';hcg=50'; b= intervalo perforado = 30'
Espaciamientoto: 80 acres(romboidal) Kh=90 md; kv=
60 md.A condiciones del reservorio:
go= 0.741; gw=1.092 gg=0.098
mo=1.11 cp; bo=1.25 RB/STB.
Capa de gas y agua de fondo están presentes, pero
un activo empuje de agua no es evidente de la
historia del reservorio.
Encontrar el máximo rate de petróleo libre de gas y
agua.
Problema 46): Datos: condiciones
presente futura
presión del reservorio 2500 2000
presión fluyente 1750 -
rate de flujo 2024 -
Eficiencia de Flujo 1.0 1.0
kro 0.8 0.75
171
Rates de producción
kabs 62.5 62.5
mo 0.5421 0.6229
bo 1.319 1.2562
re 1500 1500
rw 0.25 0.25
h 50 50
Calcular : el rate de flujo para Pwf =1500 psia y 1000
psia si la presión del reservorio cae a 2000 psia.
Solución:
(1) Cálculo de J y qmax
J = q/(Pr-Pwf) = 2024/(2500-1750) = 2.7
2024
qmax = = 4325 b/d
1-0.2(1750/2500)-0.8(1750/2500)²
(2) Cálculo de J*p
(1.8)(4325)
J*p = = 3.11
2500
172
Rates de producción
3)Calcular J*f con la ecuación:
J*f = J*p [kro/mobo]f / [kro/mobo]p
J*f = 3.11 [.75/(.6229)(1.2562] / [.8/(.5421)(1.319)]
J*f = 2.67
(4)Calcular los rates de flujo con la ecuación:
J*Pr
qo = [1 - 0.2 (Pwf/Pr) - 0.8 (Pwf/Pr)²]
1.8
Para Pwf= 1500 psia a Pr =2000 psia
2.67x2000 1500 1500
qo= [1- 0.2( ) - 0.8 ( )²] = 1186 b/d
1.8 2000 2000
Para Pwf= 1000 psia a Pr =2000 psia
2.67x2000 1000 1000
qo= [1- 0.2 ( ) - 0.8( )² ]= 2075 b/d
1.8 2000 2000
173
Formaciones Estratificadas
Arena 1 Pr1 = 2500 psi Arena 2 Pr2= 1800 psi
Pwf = 1500 psi Pwf = 1500 psi
qo = 60 b/d qo= 120 b/d
Determinar la Curva IPR Total
Solución:
q1max= 101,35 b/d J*1= 0,073 b/d/psi
q2max= 432 b/d J*2= 0,432 b/d/psi
p*wf = 1898 psi q1= 39,23 b/d q2= 39,23 b/d

174
Formaciones Estratificadas
IPR del Pozo
Pwf Arena 1 Arena 2 Total

2500 0 0 0
2000 33,24 0 0
1898 39,23 -39,23 0
1850 41,95 -20,76 21,19
1800 44,72 0 44,72
1700 50,08 42,13 92,21
1500 60 120 180
1000 80,27 277,33 357,60
500 94,06 381,33 475,39
0 101,35 432 533,35
175
176
Pozo de Petróleo: Encontrar el máximo rate estable
Pr = 2400 psi J*= 0,8 Psep= 100 psi
Pb = 1900 psi EF= 0,7
Profundidad= 5000' Línea Flujo= 2", 4000'
q o = J * (Pr  Pwf ) q o = J(Pr  Pb ) 
J * Pb  Pwf Pwf 2 
1.8(1  )  0.8 * EF (1  ) 
1.8  Pb Pb 
Pwf qo pd Pwh 0,7Pwh
2400
2150 200 120 740 518
2025 300 140 640 448
1900 400 170 600 420
1772 500,25 200 560 392
1640 599,15 230 450 315
1500 699,04 260 430 301
1350 800,37 300 300 210
1025999,72 370 120 84
Máximo Rate Estable: 700 b/d, f bean= 3/8"
177
178
Máximo Rate estable
Problema 47): Pozo de petróleo:Profundidad Total =
12,467'; fcasing = 7 5/8“;flowline = 6,560'. Presión del
reservorio= 4000 psia. Presión del separador = 150
psia.Una prueba arrojó qo = 438 b/d pwf = 3000 psia
GLR = 600 scf/stb.Para tubería de 2" x 2". Cuál es el
máximo rate estable con los gráficos de Hagedorn y
Brown? Cuál es el diámetro del bean?
Solución: de la ecuación de Vogel
____________
Pwf = 0.125 Pr[-1+81-80(qo/qomax)]
con lo cual se tiene la siguiente Tabla:
qo Pwf Eq.pies Prof. Eq.Pwh Pwh 0.7 Pwh

100 3792.3 19000 12467 6533 540 378


200 3573.9 18200 12467 5733 545 381.5
179
Máximo Rate estable
qo Pwf Eq.pies Prof. Eq.Pwh Pwh 0.7 Pwh
400 3097.8 15580 12467 3133 320 224
600 2548 12900 12467 433 60 42
Flujo horizontal: Longitud 6560'
Psep = 150 psia fLF = 2"
qo P sep. Equiv.pies Longitud Equiv.PD PD
100 150
200 150 16100 6560 22660 -
300 150 7000 6560 13560 220
400 150 4100 6560 10660 260
600 150 1800 6560 8360 340
La intersección de las dos curvas inferiores arroja un
qo máximo estable de 370 b/d con un THP de 250 psi.
De la fórmula del bean: f = 0.356 = 3/8"
180
Máximo Rate estable

181
Presión Fondo Fluyente Pozo Gas
Problema 48): Encontrar la presión de fondo
fluyente de un pozo de gas:
gg= 0.74 = 0.016 qg = 2.5 mmscf/d profundidad
pozo = 5000' fT= 2.441" THP = 1000 psia
Solución: para p2 = 1000 psia T2 = 623R gg=0.74
Ppc = 665 psia Tpc = 400R
Las funciones reducidas serán:
PR2= 1000/665 = 1.5; TR2= 623/400 = 1.56
Luego se encuentra B:
B = 667  Q² Tavg2/(d5 Ppc2) = 0.675
Pr= 1.5 Tr= 1.5 Pr= 1.5 Tr= 1.6 Pr= 1.5 B=0.675
I2 I2 I2
B=5 0.8212 B=5 0.8242 Tr=1.5 1.0328
B=0.675 x B=0.675 y Tr=1.56 I2
B=0 1.0659 B=0 1.0931 Tr=1.6 1.0568
x=1.0328 y=1.0568 I2 = 1.0472
182
Presión Fondo Fluyente Pozo Gas
De la respectiva fórmula:
I1=I2-0.01877 ggL/Tavg= 1.0472 - .01877x0.74x5000/623
I1= 0.936.Pr1 lo encontramos con el uso de Tablas:
B=0 Tr = 1.5 B=0 Tr = 1.6 B=0 I=0.936
I'=0.9113 Pr=1.8 I'=0.9825 Pr=1.7 Tr=1.5 Pr=1.7496
I1=0.936 x I1=0.936 y Tr=1.56 P'r1
I"=0.9603 Pr=1.7 I=0.9325 Pr=1.8 Tr=1.6 Pr=1.793
x=1.7496 y = 1.793 P'r1 = 1.7756
B=5 Tr = 1.5 B=5 Tr = 1.6 B=5 I=0.936
I1=0.936 x I1=0.936 y Tr=1.5 Pr=0.9207
I'=0.9215 Pr=1.0 I'=0.9233 Pr=1.0 Tr=1.56 P'r1
I"=0.9032 Pr=1.1 I"=0.9052 Pr=1.1 Tr=1.6 Pr=0.9298
x=0.9207 y=0.9298 P'r1= 0.9262
Luego:B=0 Pr = 1.7756
B=0.675 Pr1
B=5 Pr = 0.9262
Pr1= 1.6609; P1 = Pr1x Ppc = 1.6609x665 = 1104.5 psia
183
Rate Reservorio bajo saturado
Problema 49): Datos: PR = 4000 psia Pb = 3000 psia
qo = 200 b/d para Pwf= 2000 psia
a)Determinar el qomax, si la ecuación de flujo para la
zona saturada es q = C(Pb²-Pwf²).
b)Determinar qo para pwf = 2500 psia, con la misma
ecuación de flujo.
Solución:a) Para la zona saturada q'max = CPb2
C= q'max/Pb²
La ecuación de flujo en la zona saturada será:
q = q'max(1-Pwf²/Pb²)
dq/Pwf = -2 q'maxPwf/Pb²
Cuando Pwf = Pb
J = 2 q'max/Pb
q'max = JPb/2
Ahora le ecuación de flujo es:
q = JPb (1-Pwf²/Pb²) / 2
qtest = qb+ qsat
qtest = J(Pr-Pb) + JPb(1-Pwf²/Pb²)/2
184
Rate Reservorio bajo saturado
Despejando J:
qtest
J=
(Pr-Pb)+[Pb(1-Pwf²/Pb²)/2]
200
J= = 0.1091
(4000-3000)+[3000(1-2000²/3000²)/2]
qmax= qb + q'max = 0.1091(4000-3000)+0.1091(3000/2)
qmax= 109.1 + 163.65 = 272.75 b/d.
b)
qo
J= = 0.1091
(4000-3000)+[3000(1-2500²/3000²)/2]
qo = 159.1 b/d
185
Rate Máximo estable
Problema 50): Pozo de petróleo Pr = 3500 psia
Ec. de influjo:q = 0.424[Pr²-Pwf²)/1000]0.825;flowline 2"x
3000' y tubing 2“x7000';presión separador= 100 psi;
GLR = 600 scf/stb
a)Bean al rate máximo estable sin menoscabo de la
performance de influjo.
b)Cuál es el rate con bean de ¼",el rate es estable?
Solución:
Pwf = [Pr²-(q/0.424)1/0.825x1000]½
Para flujo vertical GLR= 600 scf/stb øTP= 2"
qo Pwf Eq.pies Prof. Equiv.Pwh Pwh 0.7 Pwh
100 3391 17700 7000 10700 1250 875
200 3242 17200 7000 10200 1270 889
400 2866 14800 7000 7800 1020 714
600 2378 12350 7000 5350 710 497
700 2074 11000 7000 4000 550 385
800 1704 9400 7000 2400 330 231
900 1213 7200 7000 200 40 28
186
Rate Máximo estable
Flujo horizontal: Longitud 3000'
Psep = 100 psia øLF = 2"
qo P sep. Equiv.pies Longitud Equiv.PD PD
200 100 7200 3000 10200 120
400 100 2200 3000 5200 170
600 100 1000 3000 4000 230
700 100 800 3000 3800 260
800 100 700 3000 3700 295
900 100 600 3000 3600 315
Para no menoscabar la performance de influjo:
(Pr- Pwf)/Pr  0.5
La intersección de las dos curvas del pD es a qo=770
b/d y un Pwf de 1824.3
(3500-1824.3)/3500 = 0.478  0.5
Por tanto es un rate adecuado.
Del gráfico THP=400 psi
De la fórmula del bean f = 0.47"  ½"
187
Rate Máximo estable

188
Rate Máximo estable
b) Para bean de ¼"
De la ecuación del bean
THP = 1.7441 q
Intersectando esta recta con la curva del THP
qo = 500 b/d <770 b/d por tanto el rate es estable.
Problema 51):Gradiente de presión en un punto de la
tubería vertical, con los siguientes datos:
Presión = 1200 psia. Temperatura del reservorio =
150F;gg = 0.65;WOR = 1; GOR = 1200 scf/stb; qo =
200 b/d;API del petróleo = 35
Flujo anular ø casing = 5½" ø tubos = 2"
Solución:
WOR = 1; GLR =GOR/(1+WOR) = 1200/2 = 600 scf/stb
Por ser flujo anular se trabaja con el diámetro
equivalente
189
Flujo anular
Para la 1a. potencia de = di - do
di = DI del casing --> di = 4.974"=0.4145'
do = DO del tubing --> do = 2.375"=0.1979'
de = 4.974-2.375 = 2.6" = 0.217 pies
Para la 2a. potencia
Ap = p(di2-do²)/4 = 0.10418 pies2
Para la 5a. potencia
d5 = (di+do)²(di-do)3= 3.81E-03
método de Hagedorn y Brown:
Solución:(1)Determinar la masa por stbL.
m
=go(350)[(1/(1+WOR)]+(350)[(WOR/(1+WOR)]+.0764(G
LR)(gg)
m= 148.75+125+29.8 = 353.546 lbm/stbL
(2)Calcular el rate de flujo de masa w
w = (353.546)(400) = 141,418.4 lbm/d.
(3)Cálculo de la densidad de la fase liquida:
190
Flujo anular
go(62.4)+(1/5.615)(0.0764)(Rs)(gg)
rL =[ ][(1/(1+WOR)]+
bo
+ {(gw)(62.4) [(WOR/(1+WOR)]}
donde Rsavg =234 scf/stb boavg = 1.136
rL = 24.26 +31.2 = 55.46 lb/pie3
(4)Cálculo de la presión promedio:
pavg = 1200 psia
(5)Encontrar z a p y T promedio. z = 0.88
(6)Calcular la densidad promedio de la fase gas:
rg = 2.7 p gg /(Tz) = 3.92 lb/pie3
(7)Cálculo de mo
mo= 3.6 cp mo = 1.9 cp
(8)Cálculo de mw
De la Fig. 2.9 mw = 0.49 cp
(9)Cálculo de la viscosidad liquida de la mezcla
mL = mo [(1/(1+WOR)] + ?w [(WOR/(1+WOR)]
mL = 0.95 + 0.245 = 1.195 cp
191
Flujo anular
(10)Asumiendo constante la tensión superficial en el
punto de presión, calcular la tensión superficial de la
mezcla: L = o [(1/(1+WOR)] + w [(WOR/(1+WOR)]
o = 27.5 dinas/cm w = 70 dinas/cm
L = 13.75 + 35 = 48.75 dinas/cm
(11)Calcular el Número de viscosidad del líquido.
NL = 0.15726 mL(1/rL3)¼ = 0.00373
(12)De la Fig. 2.56 determinar
CNL = 0.0021
(13) Calcular el área del tubing
Ap = pd²/4 = 0.10418 pies²
(14) Calcular la velocidad superficial liquida vsL
pies/seg
(5.615)(qL)
vsL= {boavg[(1/(1+WOR)]+bw[(WOR/(1+WOR)]}
(86400)(Ap)
vsL= 0.266 pies/seg
(15) Calcular el Número de velocidad del líquido:
NVL = 1.938 vsL (rL/)¼ = 0.53 192
Flujo anular
(16) Calcular la velocidad superficial del gas vsg
pies/seg
(qL){GLR-Rs [(1/(1+WOR)]} 14.7 Tavg
vsg= ( )( ) zavg
(86400)(Ap) pavg 520
vsg = 0.27 pies/seg
(17) Calcular el Número de velocidad del gas:
NVg = 1.938 vsg(rL/)¼ = 0.54
(18) Calcular el Número del diámetro:
ND = 120.872 d(rL/)½ = 26.43
(19) Calcular la función de correlación del holdup:
[(NLv)/(Ngv)0.575] (P/Pa)0.1 ( CNL/ND )= 0.00012
(20) De la Fig. 2.34 HL/¥ = 0.38
(21) Cálculo del parámetro factor secundario de
corrección
Ngv NL0.38
= 0.00006
ND2.14
193
Flujo anular
(22) Obtener ¥ de la Fig. 2.57
¥ = 1.00
(23) HL = (HL/¥)(¥)= 0.38
(24) Cálculo del colgamiento sin deslizamiento:
vsL 0.266
L = = = 0.5
vsL + vsg 0.266+ 0.27
Como L > HL no se sigue con Hagedorn y Brown.
Metodo de Poettmann y Carpenter:
1) rm = rL L + rg (1-L) = 29.5 lb/ft3
2) Cálculo de f'
f' = ( 1.4737x10-5 qoMo/d) = (10.166)
f' = 0.065
3) Determinación de la gradiente:
dP == 1_ rm + __f'(qoMo)²_____
dh total 144 7.413x1010rmd5
dP == 0.2061 psi/pie
dh total
194
Flujo Direccional
Problema 52) Beggs y Brill para flujo direccional. El problema
anterior con un ángulo de 30 con la vertical.
9) Factor de corrección por inclinación del holdup líquido:
¥ = 1 +C[sen(1.8q) - 0.333 sen3 (1.8q)] = 1.09
10) Calcular el holdup líquido y la densidad de dos fases
HL(60) = HL(o)¥ = 0.485x1.09 = 0.5287
rtp = 61.95x0.529 + 2.47(1-0.529) = 33.915 lb/pie3
11) Calcular la razón del factor de fricción
£ 0.38
y=[ ]=[ ] = 1.374
HL60 2 0.529²
ftp lny
= exp [ ]
fns {-0.0523+3.182lny-0.8725[lny]²+0.01853[lny]4}
(ftp/fns) = exp(0.3646) = 1.44
12) Calcular el factor de fricción sin deslizamiento
1
fns = = 0.026
[2log(NRens / (4.5223logNRens-3.8215))]²
195
Flujo Direccional
13) Calcular el factor de fricción de dos fases
ftp
ftp = fns( ) = 0.026 (1.44) = 0.0375
fns
14) Calcular Dz
rtpvm vsg 33.195x8x4.96
144Dp [1 - ] (144)(500) [1 - ]
gc p 144 32.17x764.7x144
Dz = =
ftp Gm vm 0.0375x202.37x8
rtpsenq + 33.195x0.866 +
2 gc d 2x32.17x0.16625
Dz = 2090.8‘
Problema 53): Beggs y Brill para flujo horizontal. Los mismos
datos con ángulo de 0 con la horizontal.
10) Calcular la densidad de dos fases
rtp = 61.95x0.485+2.47(1-0.485) = 31.32 lb/pie3
11) Calcular la razón del factor de fricción
£ 0.38
y=| |=| | = 1.632
HL0 2 0.485²
196
Flujo Horizontal
ftp lny
= exp [ ]
fns {-0.0523+3.182lny-0.8725[lny]²+0.01853[lny]4}
(ftp/fns) = exp (0.3773) = 1.4584
12) Calcular el factor de fricción sin deslizamiento
1
fns = = 0.026
[2log(NRens/(4.5223logNRens-3.8215))]²
13) Calcular el factor de fricción de dos fases
ftp
ftp = fns( ) = 0.026(1.4584) = 0.03792
fns
14) Calcular DL
rtpvm vsg 31.32x8x4.96
144Dp [1 - ] (144)(500) [1- ]
gc p 144 32.17x764.7x144
=
ftp Gm vm 0.03792x202.37x8
2 gcd 2x32.17x0.16625
DL = 12541.36'
197
Separador Vertical de 2 fases
Problema 54): Diseño de un separador vertical de dos fases:
Datos: Rate de flujo 10 MMscfd a 0.6 sp.gr 2000 b/d petróleo
40API.Presión de operación = 1000 psia. Temperatura de
operación= 60F
Solución:(1) Cálculo de k
(gg p)/ T = (0.6)(1000)/(520) = 1.1538
De la Fig. 4.10 k = 0.3
(2) Cálculo de la capacidad de gas de confinamiento
T z Qg
d² = 504 k ; z = 0.84
p
(520)(.84)(10)
d² = 504 = 25.7"
1000
(3) Cálculo de la capacidad de líquido de confinamiento
trQl
d²h =
0.12
4) Se calculan combinaciones de d(diámetro mínimo para la
separación del gas) y h(altura del volumen líquido) en pulgadas.
Tabla 4-1
198
Separador Vertical de 2 fases
5)Se calcula longitudes seam to seam(costura a costura)
Lss = (h+76)/12 o Lss = (h+d+40)/12
6) Calcular el radio de slenderness (consistencia)(12 Lss/d)
Escoger en el rango de 3 a 4 es lo más común.
Tabla 4-1
tr= 3 min tr= 2 min tr= 1 min
d" h' Lss' 12Lss d" h' Lss' 12Lss d" h' Lss' 12Lss
d d d
24 86.8 13.6 6.8 24 57.9 11.2 5.6 24 28.9 8.7 4.4
30 55.6 11.0 4.4 30 37.0 9.4 3.8 30 18.5 7.9 3.2
36 38.6 9.6 3.2 36 25.7 8.5 2.8 36 12.9 7.4 2.5
42 28.3 8.7 2.5 42 18.9 7.9 2.3
7) Graficar los resultados y escoger un tamaño razonable con
un diámetro grande que esté determinado por la capacidad de
la línea de gas. En la Fig.4-12 un separador de 36"x10'
proporciona ligeramente más de tres minutos de retención, uno
de 30"x10' cerca de dos minutos y 36"x7'6" un minuto de
retención.
199
Separador Vertical de 2 fases

200
Separador Horizontal de 2 fases
Problema 55): Diseño separador horizontal de dos fases
Datos: Rate de flujo 10 MMscfd a 0.6 sp.gr
2000 b/d petróleo 40API. Presión de operación = 1000 psia.
Temperatura de operación = 60F
Solución:
(1) Cálculo de k: (gg p)/ T = (0.6)(1000)/(520) = 1.1538
De la Fig. 4.10 k = 0.3
(2) Cálculo de la capacidad de gas de confinamiento
TzQg
d Leff = 42 | | K ; z = 0.84
P
(520)(0.84)(10)
d Leff = 42 | | (0.3) = 55.04
1000
3) Se computan longitudes seam to seam (costura a costura) para
varios d (Tabla 4-2)
Lss = Leff + d/12
Tabla 4-2
d" Leff' Lss'
16 3.44 4.8
20 2.75 4.4
24 2.29 4.3
201
Separador Horizontal de 2 fases
4) Se determina la capacidad del líquido por la ecuación:
tr Q l
d²Leff = ------
0.7
5) Se calculan combinaciones de d(diámetro mínimo para la
separación del gas en pulgadas) y Lss seam to seam(costura a costura
en pies) para varios tr.Tabla 4-3
Tabla 4-3
tr=3 min tr=2 min tr=1 min
d" Leff' Lss' 12Lss d" Leff' Lss' 12Lss d" Leff' Lss' 12Lss
d d d
24 14.9 19.8 9.9 24 9.9 13.2 6.6 20 7.1 9.5 5.7
30 9.5 12.7 5.1 30 6.3 8.4 3.4 24 5.0 6.6 3.3
36 6.6 8.8 2.9 36 4.4 5.9 2.0
42 4.8 6.5 1.8
6) El radio de slenderness (consistencia)=(12 Lss/d)
Escoger en el rango de 3 a 4 es lo más común.
7) Graficar los resultados y escoger un tamaño razonable con una
combinación de diámetro y longitud ambos relacionados con la
capacidad de las dimensiones de las líneas de gas y líquido. En la Fig.
4-13 un recipiente de 36"x 7'6" o 30"x10' proporcionan cerca de 2.5
min. de retención. Uno de 30"x7'6" algo menos de 2 min. de retención
202
Separador Horizontal de 2 fases

203
Separador Vertical de 3 fases
Problema 56): Diseño de un separador vertical de tres fases: Datos: qo
= 5000 b/d de petróleo.
qw = 3000 b/d de agua ; qg = 5 MM scfd
Po = 100 psia; API petróleo = 30
gw = 1.07; gg = 0.6
(tr)o=(tr)w=10 min; mo = 10 cp
Solución:
1) Calcular la diferencia en gravedades específicas.
D sp.gr = 1.07-0.876 = 0.194
2) Calcular el mínimo diámetro para satisfacer la capacidad de gas
confinado. ( De acuerdo al criterio de dos fases).
dmin = 45.1"
3) Calcular el mínimo diámetro para el asentamiento de gotas de agua.
d2min = 0.027 (qo mo/ D sp.gr)
d2min = 0.027 (5000x10/0.194) = 83.4"
4) Retención del líquido confinado
(tr)o(qo) (tr)w(qw)
ho = hw =
0.12 d² 0.12 d²
10 (qo + qw)
ho + hw =
0.12 d² 204
Separador Vertical de 3 fases
5) Computar combinaciones de d y ho + hw para diámetros
mayores que dmin. Tabla 5-1
Tabla 5-1
Capacidad Separador Vertical de 3 Fases
Diámetro vs Longitud para Tiempo de
Retención de Confinamiento = 10 min.
d" (ho+hw)" Lss' 12Lss/d
84 94.4 14.2 2.0
90 82.3 13.2 1.8
96 72.3 12.4 1.5
102 64.1 11.7 1.4
6) Cálculo longitudes seam to seam.
Lss = (ho+hw+76)/12 o (ho+hw+d+40)/12
7) Calcular el radio de slenderness (consistencia)(12 Lss/d)
Escoger en el rango de 1.5 a 3 es lo más común. Tabla 5-1
8) Si es necesario repetir las etapas 6 a la 10 para varios
tiempos de retención y graficar como se hizo para el cálculo de
dos fases.
9) Escoger un razonable tamaño. Uno de 90"x15' o de 96"x12'6"
son razonables tamaños.
205
Separador Vertical de 3 fases

206
Separador Horizontal de 3 fases
Problema 57): Diseño De Un Separador Horizontal De Tres Fases
Datos:Los mismos que el vertical.
1) Calcular la diferencia en gravedades específicas.
D sp.gr = 1.07-0.876 = 0.194
2) Chequeo para la separación de gas. Ver procedimiento para flujo de
dos fases.
d Leff = 143
3) Computar combinaciones de d y Leff para separación de gas Tabla
5-2. Debido a los valores bajos de Leff, la capacidad del gas no
gobierna.
4) Calcular el máximo espesor de la capa de petróleo
(ho)max = 320 [(tr)o(?sp.gr)]/?
(ho)max = 320 [(10)(0.194)]/10 = 62.1
5) Calcular el máximo diámetro para el espesor de la capa de petróleo
confinada:
Aw qw(tr)w
= 0.5
A qo(tr)o+qw(tr)w
Aw (3000)(10)
= 0.5 = 0.1875
A (5000)(10)+(3000)(10)
Con (Aw/A) entrar a la Fig.5.7 y hallar Z = 0.257
dmax = (ho)max/ Z = (62.1)/(0.257) = 241.6" 207
Separador Horizontal de 3 fases
6) Retención del líquido confinado.
d2 Leff = 1.42 [qo(tr)o+qw(tr)w]
d2 Leff = 1.42 [(10)(8000) = 113,600
7) Computar combinaciones de d y Leff.Tabla 5-3
8) Se computan longitudes seam to seam(costura a costura). Tabla
5-3.
Lss = Leff/0.75 o Lss = Leff +(d/12)
9) Calcular el radio de slenderness (consistencia)(12 Lss/d)
Escoger en el rango de 3 a 5 es lo más común.
Tabla 5-1
Capacidad Separador Horizontal de 3 Fases
Diámetro vs Longitud para Tiempo de
Retención de Confinamiento=10 min.
d" Leff' Lss' 12Lss/d
60 31.6 42.1 8.4
72 21.9 29.2 4.9
84 16.1 21.5 3.1
96 12.3 16.4 2.1
108 9.7 13.0 1.4
10)Graficar los resultados y escoger un razonable tamaño que cumpla
la capacidad confinada de gas o de la capa de petróleo. De la Fig. 5-8
son posibles de escoger 90"x20', 96'x17'6" y 102
208
Separador Horizontal de 3 fases

209

Вам также может понравиться